You are on page 1of 91

SCH 101:

INTRODUCTION TO
PHYSICAL CHEMISTRY
STUDENT NOTES

BY:
MR. MWITA JAMES
UNIT1: THE GASEOUS STATE
1.1: Introduction

The State of Matter

Matter exists in any one of the following state (phases); solid, liquid and gas. The three are
clearly convertible as in ice, water and steam, which represent the same substance in different
phases. But what makes them different? In the solid state, particles are fixed in a uniform
manner in definite positions in a crystal lattice by strong forces operating between them. As a
consequence, solids are fairly rigid, have a definite shape and volume and resist compression or
distortion. Only vibrational motion is allowed for the solid particles. This vibrational motion is
dependent on the temperature, thus the higher the temperature the longer the distance of
vibration hence the weaker the forces between the particles. At a given temperature these forces
become so weak that the solid structure breaks down, i.e. melts to form a liquid.

In the liquid state the particles can move freely i.e. posses translational motion, with only weak
forces operating between the particles as compared to solids, which are however not weak
enough to allow complete separation of the particles from one another. Therefore a liquid has a
definite volume but takes the shape of the container holding it i.e. no definite shape. These weak
forces depend on the temperature too. Thus the higher the temperature the faster the particles
become and therefore the weaker the forces holding them together. As in solids, at a certain
temperature the forces become so weak that the liquid state breaks down, i.e. boils to form a gas.

In the gaseous state the restraining forces of attraction have been overcome completely such that
the particles move in a completely random manner at high speeds. Hence a gas has no definite
shape or volume; fills the container holding it and is thus highly compressible. The sensitivity to
changes in volume with changes in pressure and temperature is therefore:
Solid<Liquid<<Gas

Clearly the importance of the relationship between volume, temperature and pressure is only
important for the gaseous state and will be our subject in this chapter.

1.2: LEARNING GOALS


By the end of this unit you should be able to:
• Describe how a gas responds to changes in pressure, volume, temperature, and the
quantity of gas.
• Use the ideal-gas equation to solve for one variable (P,V,n,orT) given the other
three variables or information from which they can be determined.
• Use the gas laws, including the combined gas law, to calculate how one variable
of a gas (P,V,n,orT) responds to changes in one or more of the other variables.
• Calculate the partial pressure of any gas in a mixture, given the composition of
that mixture.
• Calculate the mole fraction of a gas in a mixture, given its partial pressure and the
total pressure of the system.
• Describe how the relative rates of effusion and diffusion of two gases depend on
their molar masses (Graham’s law).
• Explain the origin of deviation shown by real gases from the relationship
PV/RT=1 for a mole of an ideal gas.
• Cite the general conditions of P and T under which real gases most closely
approximate ideal-gas behaviour.
• Explain the origins of the correction terms to P and V that appear in the van der
Waals equation.
• Describe how molecular velocities are distributed in a sample of gas

1.3: THE GAS LAWS


Experiments with a large number of gases reveal that four variables are usually sufficient to
define the state or condition, of a gas: temperature, T, pressure, P, volume, V and the quantity of
the gas, which is usually expressed as the number of moles,n. The equations that express the
relationship among P, T, V and n are known as the gas laws or equations of state.

1.3.1 The Pressure-Volume Relationship: Boyle's Law


The first person to investigate the relationship between the pressure of a gas and its volume was
the British Chemist Robert Boyle (1627-1691).

Simulation of Boyle's Law

A quantity of gas is trapped in the tube behind a column of mercury. Boyle changed the pressure
on the gas by adding mercury to the tube. He found that the volume of the gas decreased as the
pressure increased. The pressure exerted on a confined gas is varied and the change in volume is
recorded. A relationship between pressure and volume is determined. Pressure is the manipulated
variable and volume is the responding variable. The temperature and amount of gas are
controlled variables.

For example, doubling the pressure caused the gas volume to decrease to one-half its original
value. Plots of pressure against volume at a constant temperature (isotherms) at two different
temperatures are shown in Fig.1.2.

Boyle’s law states that:

'At a constant temperature, the volume of a definite mass of a gas is inversely


proportional to pressure'
When two measurements are inversely proportional, one gets smaller as the other gets larger.
Thus when volume increases, pressure decreases. To perform his gas experiment, Boyle used a J-
shaped tube like that shown in Figure 1.1

1
From the figure, as volume increase pressure decrease i.e. V 
P

Boyle's law can be expressed in mathematical terms.

1
V = cons tan t x or PV = constant 1.1
P

Teaching Note: The equation PV = constant describes a hyperbola.

Figure 1.1 An illustration of Boyle’s experiment


In (a) the volume of the gas trapped in the J-tube is 60 ml when the gas pressure is
760 mmHg. When additional mercury is added, as shown in (b), the gas trapped is
compressed. The volume is 30 ml when the total pressure is 1520 mmHg
corresponding to atmospheric pressure plus the pressure exerted by 760 mm
column of mercury.
Figure 1.2 A plot of pressure against volume at constant temperature based on Boyle’s law

Hyperbolic curves obtained at T1 and T2.

1
A plot of P against , Fig. 1.3 yields a straight line, with a constant gradient given by PV.
V
Hence PV = constant which is another way of stating Boyle's Law

Fig. 1.3 Plot of P against 1/V at a constant temperature

At a given temperature, if the volume of a gas is V1 at a pressure P1 then if the volume changes,
say to V2, then the pressure will change to a new value P2 and

P1V1 = P2V2 1.2

since PV must remain constant.


Usefulness:
Calculating the volume of a gas at any required pressure, if the volume at another pressure is
known.

Sample Exercise 1.1


The volume of a gas at 1 atm pressure is 390 cm3. Calculate the volume that the gas will occupy
if the pressure is increased to 1.5 atm at constant temperature.
PV
P1V1 = P2 V2  V2 = 1 1
P2
3
P1 = 1 atm; V1 = 390 cm ; P2 = 1.5 atm
3
1atm x 390 cm
V2 =
1.5 atm
= 260 cm3

Practice Exercise 1.1


Calculate the volumes that would be occupied by a given gas at the given final pressure by the
gases whose initial volumes are given. The temperatures remain constant.

Initial volume/cm3 Initial pressure Final pressure


(a) 152 750 mm/Hg 760 mm/Hg
(b) 180 760 mm/Hg 720 mm/Hg
(c) 5250 1 atm 1.35 atm
(d) 1540 1.01 x 105 7.35 x 104 Pa

1.3.2: The Temperature - Volume Relations: Charles's Law

The early work of Charles (1787) and later by Gay - Lussac (1808) showed that, if the pressure is
kept constant, the volume of a sample of gas varies linearly with temperature. Thus doubling
temperature causes the gas volume to double. Therefore volume is directly proportional to
temperature, when temperature increases volume increases. If a volume vs. temperature graph is
plotted for gases, most lines can be interpolated so that when volume is 0 the temperature is -273
C. Naturally, gases don’t really reach a 0 volume, but the spaces between molecules approach 0.
At this point all molecular movement stops. –273C is known as “absolute zero” (no EK). Lord
Kelvin suggested that a reasonable temperature scale should start at a true zero value. He kept
the convenient units of C, but started at absolute zero. Thus, K = C + 273. 62C = ? K:
K=C+273 = 62 + 273 = 335 K . Notice that kelvin is represented as K not K.

Looking back at the temperature vs. volume graph, notice that there is a direct relationship. It can
be shown that V/T = constant

Give Charles’s law in words and as an equation.


Charles’s Law: as the temperature of a gas increases, the volume increases proportionally,
provided that the pressure and amount of gas remain constant,
V1/T1 = V2/T2

alternatively
"The volume of a fixed amount of gas maintained at a constant pressure is directly proportional
to its absolute temperature"

'The volume of a fixed mass of any gas increases by the same fraction (V) for every degree rise
in temperature’.

Thus doubling absolute temperature, say from 200 K to 400 K, caused the gas volume to double.
Mathematically, Charles's law takes the following form:

V = constant x T

The value of the constant depends on the pressure and amount of gas.
V
or = cons tan t
T

Therefore, if V0 is volume of a given quantity of gas at 00C (at a constant pressure) the volume
Vt at any temperature t 0C is given by

Vt = V0 + VV0t = V0(1+(Vt) 1.4

V, the coefficient of expansion is equal to 1/273.

Therefore,

Vt =V0 (1+t/273) 1.5

Consider then, two difference volumes V1 and V2 at temperatures t1 and t2 respectively.

Then
V1 = V0(1+t1/273) 1.6
V2 = V0(1+t2/273) 1.7
V1 T1 V1 V2
and therefore = or = 1.8
V2 T2 T1 T2

V1 273 + t1
=
V2 273 + t 2

V
We can conclude that = cons tan t . Thus an alternative way of stating Charles’ law is
T
‘The volume of a given mass of gas at constant pressure is directly proportional to the absolute
temperature, V  T.

Charles' Law Simulation

We find that if a plot of volume against temperature is made, then a straight line is obtained with
an intercept V0 at 0C, Fig. 1.4. If the line is extrapolated it intercepts the x=axis at -273 0C. If
we make this as our new origin then we obtain a new temperature scale called the Kelvin scale
and the temperatures are then referred to as absolute temperatures (T Kelvin or TK). Note also
that the gas is predicted to have zero volume at this temperature. Of course, this condition is
never fulfilled because all gases liquefy or solidify before reaching this temperature.

Point of Emphasis: Always use temperature in Kelvin in gas problems.


Figure 1.4: Plot of volume against temperature at fixed pressure

Clearly then, the absolute temperature T K is related to the temperature t 0C by


T = 273 + t

Considering now Equation1.7, then

V1 T1 V V
= or 1 = 2 1.8
V2 T2 T1 T2

V
We can then conclude that = cons tan t 1.9
T

Thus an alternative way of stating Charles law is

'The volume of a given mass of a gas at constant pressure is directly proportional to the
absolute temperature, V  T'

Therefore, plots of volume against temperature at constant pressure (Isobars) are straight lines,
Fig.1.4.
Sample Exercise 1.2

At 0 0C the volume of a given gas is 546 cm3. Calculate the volume the same gas would occupy
at 140C.

Solution:
V1 V 2
=
T1 T2
V1T2 546cm 3 x 278 K
V2 = =
T1 273 K
V1 = 546 cm3, T1 = 0C + 273 = 273 and T2 = 14C + 273 = 273 K
= 574 cm 3
Practice Exercise 1.2
Calculate the volumes, which would be, occupied at the given final temperatures by the gases
whose initial volumes and temperatures are given. The pressure remains constant.

Initial volume/cm3 Initial temperature Final temperature

864 15oC 200C

580 170C 00C

1168 292 K 273 K

516 - 150C 120C

1.3.3 The Avogadro's Law: The Quantity-Volume Relationship


As we blow a gas to a balloon, the balloon expands. The volume of a gas is affected not only by
pressure and temperature but the amount of gas as well. The relationship between the quantity of
gas and its volume follows from the work of Joseph Louis Gay-Lussac (1778-1823) and
Amadeo Avogadro.

Avogadro's law follows from Avogadro's hypothesis:

The volume of a gas maintained at constant temperature and pressure is directly proportional to
the number of moles of the gas.

That is,

V = constant x n

Thus, doubling the number of moles of gas will cause the volume to double if T and P remain
constant. Experimentally, 1 mole of a gas at a given pressure and temperature occupies the same
volume as any other gas at the same temperature and pressure. Under Standard Temperature and
Pressure (S.T.P) which are 00C (273 K) and 1 atm. pressure respectively this volume is found to
be 22.4 litres. Thus according to Avogadro's hypothesis.
'Equal volumes of all gases at the same temperature and pressure contain the same
number of molecules. Hence V  no. of moles (n) at constant temperature and
pressure'.

For 1 mole, this number is 6.023 x 1023 molecules also known as the Avogadro's number.

How can we combine the three laws so far considered to obtain an equation of state?

1.4: THE EQUATION OF STATE AND THE IDEAL-GAS EQUATION


Results of Boyle and Gay - Lussac may be combined in an expression, which represents the
relation between pressure P, Volume V and temperature T of a given mass of gas. Such an
expression is called an equation of state. The state of a system is its description, as given by
specifying its composition, and its pressure, volume and temperature.

In the preceding section, we examined three historically important gas laws. Each was obtained
by holding two variables constant in order to see how the other variables affect each other.
Using the symbol  which is read "is proportional to" we have

Boyle's law: V 1 (constant n, T)


P

Charles's law: V  T (constant n, P)

Avogadro's law: V  n (constant P, T)

We can combine these relationships to make a more general gas law:

nT
V
P ……………………………. 1.11

Consider a gas initially at a state where volume is V1, pressure, P1 and at temperature T1, if the
state changes such that the new variables are V2, P2 and T2 respectively then how will the two
state be related? If initially the temperature, T1 is kept constant and the pressure changes to P2,
the volume V1 will change according to Boyle's law to V1' i.e.

P1V1 = P2 V1' 1.12

and therefore

P1 V1
V1' = 1.13
P2

If now the pressure is kept constant at P2 and the temperature changed to T2 then the volume, V1'
changes to V2 according to Charles law, i.e.
V1' V2
= 1.14
T1 T2

Substituting value of V1' from Equation. 1.12, then


P1V1 V2
=
P2 T2 T2

or

P1V1 P2 V2
= 1.15
T1 T2

As before it is clear that

PV
= cons tan t 1.16
T

Applying Avogadro's law where for 1 mole of gas, if temperature and pressure are kept constant,
all gases occupy the same volume then

PV
=R 1.17
T

Where R is our constant in Equation. 1.16, which is the same for 1 mole of all gases, referred to
as the gas constant. Then for n moles, Equation. 1.17 becomes

PV
= nR 1.18
T

Equation 1.18 is called the Ideal Gas Equation and is obeyed by all ideal gases. The concept of
ideal gases will be made clearer later. The ideal gas equation can be utilized in determining
several other quantities involving gases e.g. Concentration (C) is defined as number of moles per
given volume i.e. n/V. Therefore since

n
P= RT 1.19
V

then

P = CRT 1.20

Also since n is given by mass of gas (m) as a fraction relative to formula mass M, i.e. m/M then
m RT mRT
n M
P= RT = = 1.21
V V MV

and also since m/V defines density,  , then

RT
M= 1.22
P

1.4.1 The Gas Constant, R


The gas constant R is a very important quantity and we need to work out its value and units. At
STP the volume of 1 mole of an ideal gas is 22.4 1 or 22.4 x 10-3 m3, the pressure is 101325 Nm2
or 1 atm while the temperature is 273 K. Hence in S1 Units

PV 101325 Nm −2 x 22.4x10 −3 m 3
R= =
nT 1 molx 273 K

= 8.314 J mol-1K-1 (J = Nm)

Therefore, alternatively, it is easier to work with litres for volume and atmospheres for pressure

PV 1 atm x 22.4 l
R= =
nT 1 molx 273 K
= 0.0821 l-atm mol-1K-1

The value of R as can be seen depends on our units of volume and


pressure

TABLE
Numerical values of the Gas Constant, R, in Various Units

Units Numerical Value

l – atm mol-1K-1 0.0821


Cal mol -1K-1 1.987
J mol -1K-1 8.314
m3-Pa mol -1K-1 8.314
L-torr mol -1K-1 62.36

Point of Emphasis:
• In working with gases, the most commonly used values of R are 0.08206 l – atm mol-1K-1
and 8.314 J mol -1K-1

• The standard condition for gas behavious (0 0C,1 atm) are not the same as the standards
conditions (STP) in thermodynamics and thermochemistry (25 0C, 1 atm).

• Any of the gas law can be derived from the ideal-gas equation.

Sample Exercise 1.3

What is the volume of 1 mole of an ideal gas at STP

Solution:
nRT
V=
P
at STP, P = 101325 N m-2, T = 273 K and R = 8.314 J mol-1 K

1mol x8.314 Jmol −1K −1x 273 K


therefore V = −2
= 0.0224Jm 2 N −1
101325 Nm
(J=Nm) hence 0.0224 m3 or 22.4 l

Practice Exercise 1.3


The density of Nitrogen gas is 1.68 g 1-1, what volume will the gas occupy at 273 and at 100
Kpa.

Repeat sample exercise 1.3 using R = 0.0821 lit atm mol-1K-1

1.5: DALTON'S LAW OF PARTIAL PRESSURES


This law is concerned with the pressure of a mixture of gases. Due to the nature of ideal gases,
in a mixture of one or more gases each gas exerts the same pressure as that which it would have
exerted if it alone occupied the container. This is referred to as Partial Pressure of the gas.

The law states that:


'The total pressure of a mixture of gases is equal to the sum of the partial pressure of
the constituent gases'.

i.e. if p1, p2 and p3 ….. are the partial pressures of the individual gases in a mixture, then the
total pressure P is given by:
P = p1 + p2 + p3 + ….. ……………………………………… 1.23

Considering the ideal gas equation we can see that;

p1V = n1RT and p2V = n2RT etc ………………………………… 1.24

Hence
(P1 + P2 + P3 ….)V = (n1 + n2 + n3 + ….)RT ………………………. 1.25

or
PV = ntRT …………………………………………………………. 1.26

Where nt is total number of moles i.e. n1+n2+n3 …..

From Equation. 1.24 and 1.26 the partial pressure p1 is given by:

n1 n
p1 = P = X1P and p 2 = 2 P = X 2 P etc. …………………………….. 1.27
nt nt
n n
X1 and X2 given by 1 and 2 respectively, are called the Mole Fraction of the respective
nt nt
gases.

Teaching Note: When explaining Dalton’s law, emphasize that each component is at the same
temperature and occupies the same volume.

Point of Emphasis: The sum of the mole fractions of a mixture must equal 1 i.e. X i =1

Sample Exercise 1.4


100 ml of oxygen at 1.75 atm and 200 ml nitrogen at 0.6 atm are passed into a vessel whose
capacity is 500 ml. Calculate the total pressure of the mixture in the vessel at the same
temperature.

Solution
From Boyle's Law we can obtain the partial pressure of each gas as volume changes to 500 ml.

P1V1 = P2V2

100 ml x 1.75 atm = 500 ml x PO 2 and 200 ml x 0.6 atm = PN 2 x 500 ml


PO 2 = 0.35 atm and PN 2 = 0.24 atm
Ptotal = PO 2 + PN 2 = 0.35 atm + 0.24 atm = 0.59 atm
alternatively

PO 2 VO 2 + PN 2 VN 2 = Ptotal Vmix

Practice Exercise 1.4


A mixture of gases at STP contains 65% Nitrogen, 15% Carbon dioxide and 20% Oxygen by
volume. What is the partial pressure of each gas in atmospheres.

1.6: THE KINETIC MOLECULAR THEORY OF IDEAL GASES


Why does a gas obey Boyle's or Charles law? To understand the nature of gases, we develop a
physical picture of the nature of the gaseous state capable of explaining the various laws of
gaseous behaviour. We propose hypothesis about the nature of the gas. Such hypothesis are
referred to as a "model" for a gas. The properties of the gas deduced from this model are then
compared to the experimental properties of the gas.

Having looked at the gas laws generally, it is important to try and understand them from the
molecular point of view. To develop our theory we need to first define precisely a gas in a
container in the form of postulates. Thus the following (postulates) will hold true for any ideal
gas for the moment at low pressure and moderate temperatures.

1. Gases are made of independent particles (atoms or molecules) moving in rapid random,
motion in straight line.

2. When the molecules/atoms collide with others or with any surface there is no loss of
velocity (momentum conserved) i.e. the collisions are perfectly elastic.

3. The particles are viewed as being relatively far apart and their volumes relatively small
(negligible) compared to the total volume of the container, e.g. the gas only occupies
about 0.03% of the total volume say for 1 mole of gas at S.T.P.

4. The molecules/atoms exert no attractive or repulsive forces between them (independent).

5. The pressure (force per unit area) is due to the impact of the molecules/atoms on collision
with the walls of the container.

It is these postulates that we want to represent mathematically in the molecular theory of gases
to obtain an equation of state.
Consider a gas contained in a cube of dimension 1 metre Figure. 1.5. Focus on a single molecule
of mass m confined in an otherwise empty container of volume V(l3)

Fig. 1.4: Gas in a container showing the movement in the three directions.

1
Although the molecules move randomly, assume that an average move in paths parallel to
3
N
each pair of opposite faces of the container. Then if N is the total number of molecules,
3
molecules move along x, y and z directions, how do we calculate the pressure, which will be the
force, exerted per square metre.

Consider a molecule of mass m, moving back and forth, along the x-direction. If u is the
u
velocity of the molecule per second then it collides with same side of the container times
2l
per second (i.e it has to travel a distance 21 meters before collision). The momentum before
collision is mu and since the collision are perfectly elastic then momentum after collision will be
the same but in different direction, i.e. -mu. The change in momentum (final momentum - initial
momentum) on collision is then -mu -mu = -2mu.

This momentum is transferred to the wall, i.e. 2mu. The rate of change of momentum which is
equal to number of collision per second multiplied by change of momentum per collision will be
2mu x u/21 = mu2/1. The rate of change of momentum is equivalent to force exerted upon
collision. Therefore for each molecule a force mu2/1 is exerted on the wall of the container.
Then the force exerted by all the molecules moving in the x-direction, N/3, is
N mu 2 mNu 2
x = ………………………………………………… 1.28
3 l 3l

Area of each face is 12 and since pressure is force per unit area, then the pressure exerted on each
phase perpendicular to the x-direction will be

force mNu 2 mNu 2


P= = = ……………………………………….. 1.29
area 3l x l2 3l3

13 is equivalent to the volume of container, hence

mNu 2
P= 1.30
3V
We should note however that all the molecules will not posses the same velocity, u, thus an
average value of u 2 is employed, hence

mN u 2
P= 1.31
3V

u 2 is called the mean square velocity and is the average of u2 for all the molecules. Since no
direction of velocity is preferred then the same pressure is exerted on each of the other phases
perpendicular to the y and z directions. Hence in general Eq. 1.32 applies for all direction.

Thus from the molecular point of view Equation. 1.31 gives the relationship between P, V and
velocity of the molecules. We should note that the quantities m, N, and u 2 will be constant say
for 1 mole of any particular gas hence Equation. 1.32 can be written as

PV = Constant ……………………………………………… 1.32

Which of course is Boyle's law defined from theory.

Kinetic Energy and Temperature


We need also to know the relationship between the kinetic energy of the molecules with
temperature. The kinetic energy k per molecule is given by

1
k= mu 2 1.33
2

Hence from Eq. 1.31


2 1 2
PV = x mu 2 x N = Nk (mu = 2 x ½ mu) 1.34
3 2 3

For n moles of a gas, N is equal to the Avogadros number, (N0), multiplied by n, i.e. n No.

hence
2
PV = nN o k 1.35
3

or

2
PV = nE k 1.36
3
N o k is the total kinetic energy per mole, Ek .

Therefore the kinetic energy is constant and since at constant pressure it is proportional to the
temperature then V  T, which is Charles law. Since experimentally PV=nRT, then from
Equation 1.37

3
Ek = RT 1.37
2

i.e. kinetic energy is the same (3/2 RT) for 1 mole of all gases at any given temperature.

Teaching Note: The average kinetic energy of a gas depends only on the temperature of the gas

Grahams Law of Diffusion


2
If Equation. 1.3.1 is solved for, the root mean square, u velocity, then

3PV 3nRT
u2 = = 1.38
mN mN

If n=1 mole then N=No and mN is equivalent to the formula weight, hence

3RT
u2 = 1.39
M

This equation, due to Graham can be utilized in determining formula mass, e.g. For two gases of
2
relative formula weight M1 and M2 and root mean square relocities u 1 and u 2
respectively,
u2 1 3RT / M1 M2
= = 1.40
u 2
2 3RT / M 2 M1

Since the rate at which a gas diffuses is inversely proportional to its density, which in turn is
directly proportional to formula mass then

u2 1 M2 d2 t
= = = 2 1.41
u 2
2 M1 d1 t1

where t1 and t2 are the times it takes for equal volumes of the two gases to diffuse under similar
conditions.

Teaching Note:
For all gases at the same temperature, the average kinetic energies are identical

At a fixed temperature, the average speed of a lighter gas is greater than that of a heavier gas.

Sample Exercise 1.5


It takes nitrogen gas 2 minutes to diffuse through an orifice. How much time will methane (CH4)
gas take to diffuse through the same orifice

Solution
If x is the time methane takes, then

MN2 2 MN2 4
=  = 2
M CH 4 x M CH 4 x

28 4 64
= 2 therefore x 2 =
16 x 28

x = 1.51 minutes

Practice Exercise 1.5


It takes a gas A, 2.3 times as long to diffuse through an orifice as the same amount of oxygen.
What is the Molar Mass of gas A.
1.7: DISTRIBUTION OF MOLECULAR VELOCITIES
We have already noted that gas molecules in a container do not move with the same speed. The
distribution of the velocities was worked out using probability theory by Maxwell and Boltzman
to yield what is referred to as the Boltzman-Maxwell distribution law. Mathematically the law
is represented as

1 dN
= 4(m / 2kT)3 / 2 e − mu / 2 kT u 1.42
N du

where k is the Boltzman-Maxwell distribution constant. The physical interpretation of this


1 dN
equation is that represents the fraction of total number of molecules having a particular
N du
velocity u. Graphically the law can be represented as below, Fig. 1.5

Fig. 1.5 Distribution of molecular speeds at two different temperatures.

Note that the curves are not symmetrical and hence the average speed is higher than the most
probable. Solving the equation for 1 mole of a gas yields the following:
2kT 2RT
Most probable velocity ump = = 1.43
m M
Since k = R/No and mNo = M

u1 + u 2 8kT 8RT
Mean velocity , u = Un = = = 1.44
N m m

3kT 3RT
Root mean square velocity, u2 = = 1.45
m M

As before equation 1.39

1.8: Real Gases


An ideal gas is one that obeys the ideal gas equation of state. But few gases are ideal. In fact if
PV is plotted against P as in Fig. 1.6, the gases deviate from ideal behaviour as pressure
increases.

Fig. 1.6: plot of PV against P.

What is the cause of these deviations? Since Boyle's law agrees with the molecular theory of
gases then the problem must lie in the postulates. Firstly the molecules are said to have no
attractive or repulsive forces between them. This is not true especially at moderate to high
pressure since these forces will exist. Due to the attractive forces, the pressure exerted by the gas
will actually be less than that predicted by the theory. We need then a correction for pressure.
Hence, since the attractive forces as well as the number of molecules striking the walls are
directly proportional to the number of molecules N, then the correction term must be
proportional to N2. The number of molecules hitting the wall will be inversely proportional to
volume, hence attractive forces will vary as 1/V2. The correction term is then found to be a/V2
for 1 mole where a is a constant. i.e. P now becomes
a an 2
P + 2 or P + 2 for n moles of gas 1.46
V V

Secondly, for 1 mole of a gas the postulate that the molecules occupy negligible volume ought to
be modified, i.e. the molecules should now be considered to occupy a volume. Therefore the
volume V must be corrected for the volume of the molecules to yield the effective volume. The
correction term for volume, b, called the co-volume, which is found to be equal to four times the
actual volume of the molecules is introduced to give the actual volume as V-b for 1 mole or V-nb
for n mole of gas.
The final equation of state then becomes

a
(P + )(V-nb)=nRT 1.47
V2

for n moles of gas.

This equation is known as the Van der Waals equation. It is found to give a better description of
the state of the gas. The constants a and b are determined empirically and are unique to the gas.

Point of Emphasis
Gases deviate from ideality at high pressures

Sample Exercise 1.6


Compare the pressure predicated for 0.8 1 of 0.5 moles CO2 at 273 K using: (a) The ideal gas
equation and (b) Van der Waals equation
a = 6.581 atm mol-2 and b = 0.056 1 mol-1

(a) ideal gas equation

PV=nRT

nRT 0.5 mol x 0.821 l atm mol −1K −1 x 273 K


P= = = 14 atm
V 0.8 l
(b) Van der Waals equation

nRT an 2 0.5 mol x 0.821 l atm mol −2 K −1 x 273 K 6.58 l 2atm mol −2
P= − 2= −
V − nb V 0.8 l − (0.5 mol x 0.056 l mol ) (0.8 l) 2

=14.52 atm – 2.57 atm = 11.95 atm

As expected the pressure in (b) is less than that in (a)

Practice Exercise 1.6


Use the Van der Waals equation to predict the pressure exerted by 100 mol of argon enclosed in
a 5 1 flask at 100 0C. Compare your results with the pressure predicted by the ideal gas law.
a=2.351 12 atm mol-2 and b=0.0398 1 mol-2.

1.9: Summary
To describe the state or condition of a gas, we must specify the variables: pressure, temperature,
volume and quantity of gas. Volume is usually measured in litres (l) and temperature in kelvins.
Pressure is defined as the force per unit area. It is expressed in SI units as pascals, Pa. (1Pa =
101325 N/m2 = 1 kg/m-s2) or more commonly in torr or in atmospheres (atm). One atmosphere
equals 101.325 kPa or 760 torr.

The ideal-gas equation, PV=nRT, is the equation of state for an ideal gas. Most gases at
pressures of about 1 atm and temperatures of 300 K and above obey the ideal gas equation
reasonably well. We can use the ideal-gas equation to calculate variations in one variable when
one or more of the others are changed. For example, for a constant quantity of gas at constant
temperature, the volume of the gas is inversely proportional to the pressure (Boyle’s law).
Similarly, for a constant quantity of gas at constant pressure, the volume of a gas is directly
proportional to temperature (Charles’s law). Avogadro’s law states that at constant temperature
and pressure the volume of a gas is directly proportional to the quantity of gas, that is, to the
number of molecules.

In gas mixtures, the total pressure is the sum of the partial pressures that each gas would exert if
it were present alone under the same conditions (Dalton’s law of partial pressures)

The kinetic-molecular theory accounts for the properties of an ideal gas in terms of a set of
assumptions about the nature of gases.

The molecules of a gas do not have the same kinetic energy at a given instant. Their speeds are
distributed over a wide range; the distribution varies with the molar mass of the gas and with
temperature.
Departures from ideal behaviour increases in magnitude as pressure increases and as temperature
decreases.

EXERCISE ONE

1. During the course of a reaction 0.25 mol of gas phase reactant becomes 0.50 moles of gas
phase product. The initial volume of the gaseous reactant is 125 cm3. What is the
volume of gaseous product?

2. 1.1 L of an ideal gas at 0.4775 atm and 12.2 0C undergoes a change in volume and
pressure to 5.3 L and 1.22 atm. What is the new temperature of the gas?

3. A helium ballon has a volume of 2 L. How many moles of helium will the ballon hold if
it is filled to a pressure of 1.01 atm at a temperature of 24 0C?. How many grams of
helium does it hold?

4. What is the density of carbon tetrafluoride at 1.0 atm and 298 K.

5. What is the pressure exerted by 100 moles of CO2 in a 22.4 L vessel at 273 K, as
calculated from the Van der Waals equation? Compare your result with the pressure
predicted by the ideal gas equation. a = 3.59 L2 atm mol-2, b = 0.043 L mol-1.

6. What is the volume at STP, of 11.0 of the gas CG4?

7. 0.50 g of neon gas and 0.50 g of argon gas are placed in a 3.0 L flask at 27 0C. What is
the partial pressure of each gas and the total pressure of the mixture of gases in
atmospheres?

8. 150 g neon and 100 g oxygen are placed in a flask at a temperature such that the total
pressure is 1.22 atm. What is the mole fraction and partial pressure of each gas?

9. At a given temperature and pressure, a certain amount of argon requires 2775 to diffuse
through a porous plug. How long will it take an equivalent amount of nitrogen to diffuse
under the same conditions?

10. Calculate the pressure exerted by 40 g of oxygen enclosed in a 1 litre flask at 25 0C.
(0=16).
UNIT 2: CHEMICAL EQUILIBRIA

2.1 INTRODUCTION
The condition in which the concentrations of all reactants and products cease to change with time
is called chemical equilibrium. Chemical equilibrium occurs when opposing reactions are
proceeding at equal rates. The rate at which the products are formed from the reactants equals
the rate at which the reactants are formed from the products. For equilibrium to occur, neither
reactants nor products can escape from the system. Chemical equilibria are of importance in
explaining a great many natural phenomena and play important roles in many industrial
processes. We will learn how to express the equilibrium position of a reaction in quantitative
terms. We will also study the factors that determine the relative concentrations of reactants and
products at equilibrium.

2.2 LEARNING GOALS


By the end of this unit you should be able to:

• Write the equilibrium expression for a balanced chemical equation, whether


heterogeneous or homogeneous.
• Numerically evaluate KC (or KP) from knowledge of equilibrium concentrations
(or pressures) of reactants or products, or from the initial concentrations and the
equilibrium concentration of at least one substance.
• Interconvert KC in to KP for a given chemical equation
• Predict how a reaction at equilibrium would shift when temperature, pressure,
and/or the concentrations of substances in the equilibrium is/are changed.

2.3 THE CONCEPT OF EQUILIBRIUM


The state of equilibrium can be defined as the condition in which all substances involved in a
chemical reaction have reached a final value of concentration.
What does this mean?

Let us consider the reaction of acetic acid with ethanol

CH3COOH + C2H5OH = CH3COOC2H5 + H2O 2.1


acid alcohol ester water

We find that, if initially only the acid and the alcohol are mixed, then the ester and water are
formed. However if it is the ester and water that are mixed the acid and alcohol are produced. It
is therefore clear that Equation 2.1 can be achieved from either end. Let us concentrate on the
acid/alcohol reaction. When the two are mixed, they start forming ester and water, at a given
rate, r1, whose concentrations start increasing. The formed products also start reacting in a
reverse reaction also at a given rate, r2. Initially r1 is greater than r2 due to the differences in
concentrations. During the course of the reaction, the rate, r1, continuously decreases while r2
increases. A point is reached when r1 becomes equal to r2. Here the forward reaction takes
place with the same rate value as the reverse reaction i.e r1 = r2. The concentration then of all four
reactants remains constant. At this point, a state of equilibrium has been achieved. We should
note that both forward and reverse reactions have NOT stopped but only that they have the same
rate value. Such an equilibrium, which is not 'stagnant', is called a dynamic equilibrium. The
final constant concentrations will not necessarily be same for any reaction.

Suppose we have the general reaction

aA + bB yY + zZ 2.2

where A, B,. Y and Z are the chemical species involved, and a, b, y and z are their coefficients in
the balanced chemical equation. According to the law of mass action, the equilibrium condition
is expressed by the equation

[Y]y [ Z]z
Kc = 2.3
[A]a [B]b

The square bracket in this equation signify molar concentration i.e., moles of substance per unit
volume. We call this relationship the equilibrium expression or the equilibrium constant
expression for the reaction. The constant KC, which we call the equilibrium constant, is the
numerical value obtained when we substitute actual equilibrium concentrations into the
equilibrium expression. The subscript c indicates that concentrations (expressed in molarity) of
the species involved are used.

Note that from the balanced chemical equation for an equilibrium reaction, we can write the
equilibrium expression even if we don’t know the reaction mechanism. The equilibrium
expression depends only on the stoichiometry of the reaction, not its mechanism.

Point of Emphasis:

• The notation KC is a reminder that the amounts of materials are expressed as a


molar concentration.
• For a particular reaction, the value of the equilibrium constant varies with
temperature.

Sample Exercise 2.1

Write the equilibrium expression for KC for the following reactions.


(a) 203(g) 3O2(g)
(b) 2NO(g) + Cl2(g) 2NOCl(g)

SOLUTION:

The equilibrium expression has the form of a quotient. The numerator contains the
concentrations of the substances on the product side of the chemical equation, each raised to a
power equal to its coefficient in the balanced equation. The denominator is similarly obtained
using the substances on the reactant side of the equation.

[O2 ]3 [ NOCl]2
Kc = (b) Kc =
[O3 ]2 [ NO]2 [Cl2 ]

Practice Exercise 2.1

Write the equilibrium constant expression for H2(g) + I2(g) 2HI(g)

2.4 THE EQUILIBRIUM CONSTANTS


Equilibrium constant can be very large or very small. The magnitude of the constant provides us
with important information about the equilibrium mixture. In order for the equilibrium constant
to be so large, the numerator of the equilibrium expression must be much larger than the
denominator. We can therefore, tell the direction of the reaction from the value of the
equilibrium constant. The following are useful guides in predicting the direction of the reaction:

• When K is greater than 1: Equilibrium lies to the right. This implies that the
formation of product(s) is favoured
• When K is greater than 1: Equilibrium lies to the left. This shows that the
formation of reactant(s) are more favoured than the product(s).

We often don't know the equilibrium concentrations of all chemical species in equilibrium.
However, if we know the equilibrium concentration of at least one species, we can generally use
the stoichiometry of the reaction to deduce the equilibrium concentrations of the other species in
the chemical equation. We will use the following procedure to do this:

1. Tabulate the known initial and equilibrium concentrations of all species


involved in the equilibrium.
2. For those species for which both the initial and equilibrium
concentrations are known, calculate the change in concentration that
occurs as the system reaches equilibrium.

3. Use the stoichiomistry of the reaction to calculate the changes in


concentration for all the other species in the equilibrium.

4. From the initial concentration and the changes in concentration,


calculate the equilibrium concentrations. These are used to evaluate the
equilibrium constant.

Example
A mixture of 5.00 x 10-3 mol of H2 and 1.00 x 10-2 mol of I2 are placed in a 5.00 L container at
448 0C and allowed to come to equilibrium. Analysis of the equilibrium mixture shows that the
concentration of HI is 1.87 x 10-3 M. Calculate KC at 4480C for the reaction. The stoichiometric
equation of the reaction is shown in equation

H2(g) + I2(g) 2HI(g) 2.4

Solution
The equilibrium expression of the above equation is given as shown in equation 2.5
[HI ] 2
Kc =
[H 2 ][ I 2 ] 2.5

First, we tabulate the initial and equilibrium concentrations of all the species in the equilibrium.
In our table we also provide space for listing the changes in concentration. As shown below, it is
convenient to use the chemical equation as the heading for the table.

In this exercise, the initial concentrations of H2 and I2 must be calculated in terms of moles per
litre, thus

5.00 x 10−3 mol


[H 2 ]initial = = 1.00 x 10− 3 M
5.00 L

1.00 x 10−2 mol


[I 2 ]initial = = 2.00 x 10− 3 M
5.00 L
Thus, the first entries in the table are:

H2(g) + I2(g) 2HI(g)

[Initial] 1.00 x 10-3 2.00 x 10-3M OM


[Change]
[Equilibrium] 1.87 x 10-3M

Second, we calculate the change in concentration of HI, using the initial and equilibrium value.
The change is the difference between the initial and equilibrium value,
1,87 x 10-3M.

Third, we use the stoichiometry of the reaction to calculate the changes in the other species. The
balanced chemical equation indicates that for each 2 mol of HI formed, 1 mol of H2 must be
consumed. Thus, the amount of H2 consumed is

 − 3 molHI  1mol H 2 
1.87x10   = 0.935 x10−3 mol H 2 / L
 L  2 mol HI 

The same line of reasoning gives us the amount of I2 consumed, which is also
0.935 x 10-3 M

Fourth, we calculate the equilibrium concentrations, using the initial concentration and the
changes. The equilibrium concentration of H2 is the initial concentration minus that consumed,
that is

[H2] = 1.00 x 10-3M - 0.935 x 10-3 M = 0.065 x 10-3 M

Likewise the equilibrium concentration if I2 is

[I2] = 2.00 x 10-3M – 0.935 x 10-3 M = 1.065 x 10-3 M

The filled-in table now looks like this:

H2(g) + I2(g) 2HI(g)

Initial 1.00 x 10-3 M 2.00 x 10-3 M OM


Change -0.935 x 10-3 M -0.935 x 10-3 M + 1.87 x 10-3 M
Equilibrium 0.065 x 10-3 M 1.065 x 10-3 M 1.87 x 10-3 M

Finally, now that we know the equilibrium concentration of each reactant and product, we can
use the equilibrium expression to calculate the equilibrium constant:

[HI ] 2 (1.87 x 10 −3 ) 2
Kc = = = 50.5
[H 2 ][ I 2 ] (0.065 x 10 −3 )(1.065 x 10 −3 )

Sample Exercise 2.2

Sulphur triioxide decomposes at high temperature in a sealed container:

2SO3(g) 2SO2(g) + O2(g). Initially the vessel is charged at 1000K with SO3(g) at a
concentration of 6.09 x 10-3M. At equilibrium, SO3 concentration is 2.44 x 10-3N. Calculate the
value of KC at 1000K.

Ans 4.07 x 10-3.

Sample Exercise 2.3


The reaction of N2 and O2 to form NO might be considered a means of "fixing" nitrogen.

N2(g) + O2(g) 2NO(g)

The value for the equilibrium constant for this reaction at 250C is KC = 1 x 10-30. Describe the
feasibility of this reaction for nitrogen fixation.
Practice Exercise 2.2

The equilibrium constant for the reaction H2(g) + I2(g) 2HI(g) varies with temperature
in the following way: KC = 794 at 298 K; KC = 54 at 700K. Is the formation of HI favoured
more at a higher or lower temperature?

2.4.1 The Direction of the Chemical Equation and the Equilibrium


Constant

Because equilibrium can be approached from either director, the direction in which we write the
chemical equation for an equilibrium is arbitrary. For example, we can represent the NO2/N2O4
equilibrium as shown in equation 2.6

N2O4(g) 2NO2(g) 2.6

[ NO]2
For the equation, we can write K c = = 0.212 (at 100o C)
[ N 2O 4 ]

We could equally well consider this same equilibrium in terms of the reverse reaction as given in
equation 2.7.

2NO2(g) N2O4(g) 2.7

For this equation, the equilibrium expression at 100 0C is then given by equation 2.8

[ N 2O 4 ] 1
Kc = 2
= = 4.72 2.8
[ NO2 ] 0.212

The equilibrium expression for a reaction written in one direction is reciprocal of the one for the
reaction written in the reverse direction.

Point of Emphasis

For the value of equilibrium constant to be meaningful, we must specify how the
equilibrium reaction is written; which species are written on the product side and
which are written on the reactant side.

Practice Exercise 2.3


For the formation of NH3 from N2 and H2, N2(g) + 3H2(g) 2NH3(g),
KP = 4.34 x 10-3 at 3000C. What is the value of KP for the reverse reaction?
(Ans. 2.30 x 102).

Practice Exercise 2.4


Calculate the concentration equilibrium constant, Kc, at 298 K for the following reactions.
(a) 1 mole acetic acid is mixed with 0.18 moles ethanol to give 0.171 moles of ester at
equilibrium.
(b) 1 mole acetic acid and 1 mole ethanol are mixed to give 0.667 moles of water at
equilibrium.

Sample Exercise 2.4


At 25 oC, when 3 moles of acetic acid are mixed with 3 moles of ethanol and allowed to attain
equilibrium, the concentration at equilibrium are found to be one mole of acetic acid, one mole
of ethanol, two moles of ester and two moles of and water. What is the equilibrium constant for
this reaction?

Solution
CH3COOH + C2H5OH = CH3COOC2H5 + H2O
[Initially]: 3 moles 3 moles 0 moles 0 moles
[Equilibrium]: 1 mole 1 mole 2 moles 2 moles

KC =
CH3COOC2H5 H 2O
CH3COOHC2H5OH
2/V 2/V
= =4
1/ V  1/ V
Thus K c = 4

We note that if Kc = 4 for the above reaction, then whatever initial proportions that we start off
with, at equilibrium the concentration will be such that this value is maintained. Note also in this
case, Kc has no units.
2.3 GASEOUS EQUILIBRIA
For gases, it is more reasonable to use pressure in place of concentration. Consider the gaseous
reaction represented in equation 2.9.
eE + fF = gG + hH 2.9
We can express the equilibrium expression in terms of pressure as shown in equation 2.10
pg  p h
K p = eG fH 2.10
pE  pF
Where p represents partial pressure of each gas at equilibrium and Kp is the equilibrium constant
in terms of partial pressures.

Practice Exercise 2.5


Under equilibrium conditions at 500 K and one atmosphere total pressure, nitrosyl chloride is 27
percent dissociated according to the reaction

2 NOCl = 2 NO + Cl2

Determine KP for this reaction at this temperature.

Sample Exercise 2.5


At 300 K and one atm pressure dinitrogen tetroxide is 20 % dissociated into
nitrogen dioxide. Calculate the equilibrium constant Kp for the reaction

N2O4 = 2NO2

If the fraction dissociated, , is 0.2 then we can summarise and work out as

N2O4 = 2NO2
[Initial] 1 0

[Equilibrium] 1- 2

At equilibrium, the total number of moles will be given by

1-+2=1+

Then, the partial pressure of N2O4 will be given by


N 2O 4 =
(1 − α )P
1+ α

Partial pressure of NO2 will thus be given by

N 2O 4 =
(2α )P
1+ α
where P is the total pressure. Then the equilibrium constant in terms of pressure
will be given by
p 2NO 2
Kp =
p N 2O4

=
(2P / 1 +  )2 = 4 2 P
l −  /1 +  (1 −  )(1 +  )
4  0.22  1atm
=
(1 − 0.2)(1 + 0.2)
= 0.167 atm

2.6 RELATIONSHIP BETWEEN KC AND KP


Pressure is another way of expressing concentration. Therefore Kc can be expressed in terms of
Kp. How can we obtain the relationship between Kp and Kc?
Let us consider the following equilibrium given in equation 2.11.

N2(g) + 3H2(g) = 2NH3(g) 2.11

The concentration equilibrium constant, Kc, is given by equation 2.12


Kc =
NH 3  2

2.12
N 2 H 2  3

The partial pressure of each gas, i, can be expressed as equation 2.13

pi = (ni/V)RT = CiRT = [i]RT


2.13

Since Kp is given by the expression 2.14

p 2NH3
Kp = 2.14
p N 2 p 3H 2

Then, substituting accordingly equation 2.13 in 2.14 gives equation 2.15


(C NH 3 RT ) 2 C 2NH 3 1 Kc
Kp = = =
C N 2 RT.(C H 2 RT ) C N 2 C H 2 (RT )
2.15
3 3 2
(RT )2

It can be seen that in this case KpKc. Equation 2.16 gives a general
relationship between Kc and Kp,

Kp= Kc (RT)n 2.16

where n is the difference between number of molecules of products and those of reactants in
the stoichiometric equation.

From equation 2.11, 2 moles of NH3 are formed by reaction of 4 moles of reactants, hence n has
a value of –(minus)2. This gives the relationship as shown in equation 2.17

K p = K c (RT )
Kc
−2
= 2.17
(RT )2
In case there is no increase or decrease in number of molecules for a given reaction, n = 0 and
Kp should be equal to Kc. Consider the reaction given in 2.18.

H2(g) + I2(g) = 2HI 2.18


Clearly, n is equal to zero, hence Kp must be equal to Kc.

Common Misconception
Students frequently arrive at the wrong sign for n. In the expression (KP = KC(RT) n),
note that KP is on the left and n = product moles of gas - reactant moles of gas.

Sample Exercise 2.6


For the equilibrium
2SO3(g) 2SO2(g)
At temperature of 1000 K, KC has the value 4.07 x 10-3. Calculate the value for KP

Solution

There are 3 moles of gaseous products (2SO2 + O2) and 2 moles of gaseous reactants. Therefore
n = 3 - 2 = 1 (Remember that  functions are always based on products minus reactants).
Given that the temperature, T, is 1000 K, the value for the ideal gas constant, R, is 0.0821 lit
atm/mol K then Kp can thus be obtained as

KP = KC (RT) n = 4.07 x 10-3 x 0.0821 x 1000


= 0.334

Practice Exercise 2.6


At 457 K and total pressure of 1 atmosphere, Nitrogen dioxide is 5 percent dissociated according
to the equation

2NO2 = 2NO +O2

Calculate the equilibrium constants Kp and Kc for this reaction.

2.7: APPROACH TO EQUILIBRIUM


We have stated that no matter what amounts of material we start off with in a reaction, the
equilibrium constant after establishment of equilibrium can be approached from either reactants
forming products or vice versa. The following are examples to illustrate this fact.

a) One mole of H2 and 1 mole of I2 are introduced into a 1-litre box at 490 0C. Calculate the
final concentrations of reactants and products when equilibrium is established given that Kc =
45.9.

Solution
If n is the number of moles of H2 that react, then

H2 + I2 = 2HI
Initial no. of moles 1 1 0
No. of moles at equilibrium (1-n) (1-n) 2n

Hence

2
 2n 
 
Kc =
HI  2
= V =
4n 2
= 45.9
H 2 I2   1 − n  1 − n  (1 − n )2
 V  V 

The result is a quadratic equation, which can be solved to yield two values of n. These are 0.772
and 1.42.

Point of Emphasis:

For the equation 0 = ax2 + bx + c, the solution to x is given by the quadratic


formula:

b 2 − 4ac
x = −b 
2a

When solving for x, especially when using the quadratic formula, it is imperative to check
if the answer obtained is reasonable. For example, solutions that yield negative
concentrations can be eliminated.

Clearly n cannot have values greater than 1 in this case hence,


n = 0.772 and therefore concentrations at equilibrium are
H2= 0.228 moles, I2 = 0.228 moles and HI = 1.544 moles.

b) If initially there were 2 moles of HI that were introduced into the box and n moles reacted,
then
H2 + I2 = 2HI
Initial no. of moles 0 0 2
n n
No of moles at equilibrium 2−n
2 2
Hence

KC =
HI  2
=
(2 − n)
2
= 45.9
H 2 I2  ( n ) x ( n )
2 2

Solving for n, we obtain n= 0.456 hence concentrations at equilibrium are H2 = I2 = 0.228, while
HI = 1.544 moles.
c) All materials may be present in different amounts initially. Here the direction of the reaction
is not obvious and one may chose whatever direction, forward or reverse to work out equilibrium
concentrations. So if the following were initially present
1 mole of H2, 2 moles of I2 and 3of moles HI

If the forward reaction is assumed and if n moles of H2 react, then

H2 + I2 = 2HI
Initial no. of moles 1 2 3
No of moles at equilibrium 1-n 2-n 3+2n
Since for every n moles of H2 that react n moles of I2 react and 2n moles of HI are produced.
Note that since the forward reaction has been assumed, if it is indeed the reverse reaction that
takes place then n can have a negative value. Hence

Kc =
(3 + 2n )
2
= 45.9
(1 − n )(2 − n )
n is found to be equal to 0.684. Therefore concentrations at equilibrium are
H2 = 0.316 I2 = 1.316 HI = 4.386 moles.
(assume the reverse reaction and confirm that the same results are obtained)

2.8: HETEROGENEOUS EQUILIBRIUM


When all reactants and products are in the same phase in a reaction, at equilibrium then we have
a homogeneous equilibrium. If however, any of either the reactants or products are in a different
phase, then a heterogeneous equilibrium exists. Consider the reaction 2.19

CaCO3(s) = CaO(s) + CO2(g) 2.19

Here both solids and a gas are involved. This is an example of a heterogeneous equilibrium. The
equilibrium constant for this equilibrium can be given in terms of activity, a, Activity is defines
as the 'effective' concentration or active mass of the substances. We thus obtain the expression
2.20
a CaO a CO 2
K= 2.20
a CaCO 3
The active mass of a solid remains constant and therefore activity of solids is conventionally
unity and that of a gas is equivalent to pressure at low pressures, hence equation 2.21
K c = a CO 2 = CO2  or K p = PCO 2 2.21

since activity is also equal to concentration at low concentrations, Kp will be proportional to


pressure of CO2 only, at any given temperature. Kp is therefore determined solely by the
equilibrium pressure of CO2 and not by the amounts of CaCO3 and CaO.
Point of Emphasis

Pure solids and pure liquids have essentially constant concentrations. In effect,
we ignore the concentrations of pure solids and pure liquids, even though they
must be physically present in the equilibrium mixture.

Sample Exercise 2.7


Write the equilibrium expression for Kc and KP for each of the following reactions:
(a) CO2(g) + H2(g) CO(g)
(b) SnO2(s) + CO(g) Sn(s) + 2CO2(g)

Solution
(a) The equilibrium expressions are
2
[CO2 ]2 PCO
Kc = and K p = 2

[CO]2 2
PCO

(Because H2O appears in the reaction as a pure liquid, it's concentration does not appear in the
equilibrium expression)
2
[CO 2 ]2 PCO
b) Kc = 2
and K p = 2
2

[CO] PCO

(Because SnO2 and Sn are both pure solids, their concentrations do not appear in the equilibrium
expression).

Practice Exercise 2.7


Write the equilibrium expression for Kc and KP for the reaction
Fe(s) + 4H2O(g) Fe3O4(s) + 4H2(g).

2.9: FACTORS INFLUENCING EQUILIBRIUM CONCENTRATIONS


A reversible reaction in a dynamic equilibrium will maintain the condition indefinitely unless
disturbed by external changes. These are usually changes that will accelerate or retard either the
forward or the reverse reaction therefore causing a change in equilibrium concentrations. These
include
(i) Addition or removal of a reactant or product i.e change in concentration.
(ii) Change in pressure / volume
(iii) Change in temperature.

These effects are summed up by the Le Chateliers Principle, which states


‘If a change in any one of the factors upon which equilibrium depends on is made to a
system in equilibrium, the equilibrium shifts in the direction which tends to nullify the effect
of the change’

Teaching Note:
Consider changing the concentration or pressure of a substance in a reaction at
equilibrium as creating a stress. The removal of something creates a "deficit stress", and
adding something creates on "excess stress". Reactions will always respond to stress by
reducing the deficit (making more of what was removed) or by reducing the excess
(consuming some of what was added). This description works, regardless of the side of
the reaction to which the stress is introduced.

Point Emphasis:

If you add a substance to a reaction at equilibrium, the reaction will shift to consume
some of the added substance. If you remove a substance from a reaction at equilibrium,
the reaction will shift to produce more of the removed substance.

2.9.1: Effect of Temperature on Equilibrium


Equilibrium concentrations are always defined at a given temperature and change if the
temperature is changed i.e. the equilibrium constant is dependent on temperature. This is
because temperature increases rate of both forward and reverse reactions but not to the same
extent. For a system at equilibrium if the forward reaction is exothermic (heat released during the
reaction) then the reverse reaction is endothermic (heat absorbed during the reaction) to the same
magnitude. Consider reaction 2.26

N 2 + 3H 2 = 2NH3 H = −5508 J Exothermic 2.26


Then for the reverse reaction given as 2.27,

2NH3 = N 2 + 3H 2 ΔH = 5508 J Endothermi c 2.27

So for the reactions 2.26, according to the Le Chartliers principle, if the temperature is increased
the equilibrium should shift in the direction that absorbs heat, i.e. the endothermic reaction 2.27
is favoured. In this case (for equation 2.26), increase in temperature would cause a shift to the
left where more N2(g) and H2 (g) are formed. This results in a decrease in Kp. Otherwise,
lowering the temperature favours formation of NH3(g) , the exothermic reaction. In this case, Kp
increases.

We can deduce the rules for the temperature dependence of equilibrium constants by applying Le
Chartelier's principle. A simple way to do this is to treat heat as if it were a chemical reagent. In
an endothermic reaction, we can consider heat as a reactant, whereas in an exothermic reaction,
we can consider heat as a product. We can summarise this as
Endothermic: Reactant + heat → products
Exothermic: Reactants → products + heat

When heat is added to the system, the equilibrium shifts in the direction that absorbs heat:

Endothermic: Increasing T results in an increase in K


Exothermic: Increasing T results in decrease in K

2.9.2: Effect of Concentration on Equilibrium


At constant temperature, the equilibrium constant remains constant whatever the concentrations
or partial pressure of reactants and products. Thus if changes are made to these concentrations,
equilibrium shift to give new equilibrium concentration but Kc and Kp must remain constant.
Consider reaction 2.22

H 2 + I 2 = 2HI 2.22

At equilibrium, the rate of reaction between H2 and I2 is the same as that of formation from the
decomposition of HI. If either the concentration of H2 or I2 is increased, the rate of reaction in
the forward direction increases causing an increase in the concentration of HI. As a result, the
rate of formation of H2 and I2 then increases until a new equilibrium is attained with new
concentrations. The result, is therefore the consumption of the added H 2 or I2 to maintain the
equilibrium constant.

Thus, if either H2 or I2 or both concentration are increased, the equilibrium shifts to the right
while if HI concentration is increased the equilibrium shifts to the left. Decreasing the above
concentrations has of course the opposite effect. Note that Kc remains constant. .

2.9.3: Effect of Pressure on Equilibrium


Consider the gaseous equilibrium 2.23
N 2 (g ) + 3H 2( g ) = 2 NH 3( g )
     
2.23
4 Moles 2 Moles

According to the Le Charteliers principle an increase in pressure should cause the equilibrium to
shift in such a way that the total pressure of the gases decreases. Note that in moving from left
to right, the total number of moles changes from 4 to 2. For gases, an increase in number of
moles leads to increased pressure, while the reverse is true for a decrease. Therefore if
external pressure is increased, the equilibrium shifts towards the side with decreased number
of moles to cause a decreases in pressure. In our case, this is to the right hand side of the
equation. This way, more NH3(g) is formed. If the external pressure decreases then the reaction
shifts to the left, where the number of moles increases (pressure increases) hence more of N 2(g)
and H2(g) are formed. From equation 2.24, Kp must remain constant,
X 2NH3 I
Kp = 3 2.24
X N2 X H2 P2
If P is increased, then the ration X2NH3/XN2X3H2 must increase. This quantity increases if XNH3
increases and (XN2 X3H2) decreases. This can be achieved by the shifting of the equilibrium to
the right. The same results are obtained above. Of course, if the number of moles is the same on
both sides of the equation, then pressure has no effect on such an equilibrium. Consider the
reaction 2.25

H 2 + I 2 = 2HI 2.25

Increase or decrease in external pressure has no effect on this equilibrium.

SUMMARY
If the reactants and products of a reaction are kept in contact, a chemical reaction can achieve a
state in which the forward and reverse reactions are occurring at equal rates. This condition is
known as chemical equilibrium. A system at equilibrium does not change with time. For such a
system we can define an equilibrium constant, K. At equilibrium, the equilibrium constant
equals the equilibrium expression, which is given by the product of the concentrations of the
product(s), each raised to the power of its coefficient in the balanced chemical equation, divided
by products of reactant(s)’ concentrations. The equilibrium constant changes with temperature
but it not affected by changes in relative concentrations of any reacting substance or by pressure
or the presence of catalysts. In heterogeneous equilibria, the concentrations of pure solids or
liquids are ignored while writing the equilibrium expression.

In the case of molar concentrations, we label the equilibrium constant as KC. if the concentration
units are in terms of pressures, we use KP. The constant KC and KP are related by the equation:
KP = KC (RT) n. A large value for KC or KP indicates that the equilibrium mixture contains
more products than reactants. A small value for the equilibrium constant means the equilibrium
lies toward the reactant side.

Le Chatelier's principle states that if we disturb a system that is at equilibrium, the equilibrium
will shift to minimize the disturbing influence. The effects of adding (or removing) reactants or
products and of changing pressure, volume, or temperature can be predicted using this principle.
Exercise Two
o
1. For the reaction of an alcohol with an acid at 50 C, the concentration of each substance at
equilibrium is acid = 1.1 M, alcohol = 1.8 M, Ester = 2.6 M and H2O = 1.9 M. What is Kc for
this equilibrium?
o
2. Kc = 0.050 at 2200 C for an equilibrium of the reaction
N2(g) + O2(g) = 2NO(g)
o
If 0.50 M of each gas in the equilibrium is introduced into a flask at 2200 C, in which
direction will the reaction proceed for equilibrium to be attained?
o 3
3. At 250 C, Kc = 3.2 x 10 for the equilibrium of the reaction
PCl3(g) + Cl2(g) = PCl5(g)
What is Kp value of this reaction at this temperature?
10 -1
4. At 500 K, Kp = 2.5 x 10 atm for the equilibrium shown below. What is the corresponding
Kc value at this temperature?
2SO2(g) + O2(g) = 2SO3
5. Given the following reaction

2ICl = I2(g) + Cl2(g) Kc = 0.10

Determine the equilibrium concentration of each of the substances when ICl initially at 0.7
M is allowed to come to equilibrium.
6. At another temperature the reverse equilibrium in question 5 has Kc = 0.52. What will be the
equilibrium concentrations of all the substances in the equilibrium when I2 and Cl2 both
initially at 0.01 M concentrations are allowed to come to equilibrium?
7. For the reaction

I2(g) = 2I(g),

Kc = 6.8 x 10-2 M. What are equilibrium concentrations of I and I2 if I2 initially at 3.5 M is


allowed to come to equilibrium.
8. Consider the equation below

2HI(g) = H2(g) + I2(g)

At equilibrium the following are the concentrations, HI = 0.21 M, H2 = I2 = 0.017 M. If more


HI is added to make its concentration 0.46 M. What will be the new concentrations when
equilibrium is established and in what direction has the original equilibrium shifted?
9. For the equilibrium
2NO(g) + O2(g) = 2NO2(g),
o
Kp = 140 atm-1 at 327 C. Calculate Kp at 527 K given that H = -116 KJ mol-1
10. Kc at 25 oC for the reaction in question 9 is 4. Is the equilibrium exothermic or
endothermic?
ACID - BASE EQUILIBRIA
3.1: Introduction

Acids and bases are important in numerous chemical processes that occur around us, from
industrial processes to biological ones, from reactions in the laboratory to those in the
environment. We will see how to describe equilibria involving acid and bases quantitatively in
this section

Acids and bases may be taken as substances that increase the concentration of H+(aq) and OH-(aq)
respectively. Likewise, if the solution contains more OH- than H+ ions, the solution is basic.

Why are these two ions of such major importance in our discussion of aqueous chemistry? Is
there a relationship between the concentrations of these ions in aqueous solutions? We shall also
broaden our discussion to include additional equilibria involving slightly soluble salts.

3.2: Learning Goals


By the end of the lesson, you should be able to:
• Explain what is meant by the autoionisation of water, and write the ion product constants
expressions for this process
• Define pH, calculate pH on the knowledge of [H+] and/or [OH-] and as well perform
reverse operation
• Identify the common strong acids and bases and calculate the pH of their aqueous
solutions given their concentrations
• Calculate the pH of weak acids and bases solutions in water given their concentrations
and their respective Ka and Kb values and vise versa
• Predict whether a particular salt will be basic, acidic or neutral
• Describe how a buffer solution of a particular pH is made and the reactions that allow it
to control pH
• Calculate the change in pH of a sample of buffer solution of known composition caused
by addition of a small amount of strong acid or base.
• Calculate the concentration of each species present in a solution formed by mixing an
acid and a base
• Describe the form of the titration curves for titration of a strong acid by a strong base, a
weak acid by a strong base or a strong acid by a weak base.
• Set up the expression for the solubility-product for a salt
• Calculate Ksp from solubility data, and solubility from the value of Ksp.
• Calculate the effect of an added common ion on the solubility of a slightly soluble salt

3.3: Ion Product of Water


Pure water ionizes to a very small extent according to equilibria given in equation 3.1
H2O(l) H+(aq)+ OH_(aq)
3.1
We call this process the auto ionization of water. At room temperature, only about one out of
every 108 molecules is ionized at any given instant No individual molecule remains ionized for
long; the equilibria are extremely rapid

The equilibrium expression for the auto ionization is given in equation 3.2

[H + ][OH − ]
K= 3.2
[H 2 O]

But concentration of water remains effectively constant as K is very small hence equation 3.3

K[H2O] = Kw = [H+][OH] 3.3

The product of the two constants, K and [H2O], defines a new constant, Kw. This important
equilibrium is called the ion-product constant for water and has a value of 1 x 10-14 mol2l-2 at 25
°C. Kw must remain constant for solutions in water. Any addition of H+ or OH- from any other
source constitutes a stress causing a shift in equilibrium according to Le Charterer's principle.
Even bases will have associated with them a a certain [H-] dependent on [OH-] and Kw.
Therefore, for pure water or a solution that yields neither excess OH- nor H" ions, equation 3.4
will hold

Kw = [OH-][H+] = 10-14 3.4


Hence equation 3.5

[H+] = [OH-] = 10-7 mole l-1 3.5

A solution for which [H+] ≈ [OH-] is said to be neutral. In most solutions, H+ and OH-
concentrations are not equal. As the concentration of one of these increases, the concentration of
the other must decrease so that the ion product equals 1.0 x 10-14. In acidic solutions, [H+]
exceeds [OH-]. In basic solutions, the reverse is true: [OH] exceeds [H4].

Teaching Note: Other liquid solvents, such as NH3, also undergo autoionization

Point of Emphasis: The ion-product constant, Kw, is unaffected by whether a solution is


acidic or basic. An acidic solution has [H+] greater than [OH-], whereas

a basic solution has [OH-] greater than [H+]. At 25 °C, the product
[OH-][H+] is always 10-14.

Point of Emphasis: Because Kw varies slightly with temperature, the concentration of [H+]
and [OH-] in a neutral solution also vary slightly with temperature.
SAMPLE EXERCISE 3.1
Calculate the values of [H+] and [OH-] in a neutral solution at 25°C.

SOLUTION
By definition, in a neutral solution, [H+] equals [OH-]. Let us call the concentration of
each of these species in neutral solution x
x2 = 1.0 x 10-14
x = 1.0 x 10-7 = [OH-] = [H+]
In an acid solution, [H+] is greater than 1 .0 x 1 0 -7 M; in a basic solution [H+] is less
than 1 .0 x 1 0 -7 M

PRACTICE EXERCISE 3.1


• Calculate the concentration of H*"^ in (a) a solution in which [OH*] is 0.010 M
(b) a solution in which [OH] is 2.0 x 10* M.
• Calculate the concentration of OH(«$ in a solution in which
a) [H+] = 2 x 10-6M
b) [H+] = [OH-]
c) [H+] = 10-2 M

3.4: The pH Scale


The concentration of H+(aq) in an aqueous solution is usually quite small. For convenience, we
therefore usually express [H+] in terms of pH, which is defined as the negative logarithm in base
10 of [H+]. This is expressed in equation 3.6

pH = -log10 [H+] 3.6

For a neutral solution at 25°C, [H+] = 1.0 x 10'7M. The pH of such a solution is calculated as
shown in equation 3.7

pH = -log (1.0 x 10'7) =-(-7.00) = 7.00 3.7


The pH of a neutral solution is 7.00.

Table 3.1 summarizes the various solutions in terms of acidic, basic or neutral with reference to
the OH-, [H+] and pH

Table 3.1: Relationship between pH, [OH-] and [H+]


Solution type [H+] [OH-] pH Value
Acidic >1 x 10 <1 x 10-7 <7
-7

Neutral 1 x 10-7 1 x 10-7 =7


Basic <1 x 10-7 >1 x 10-7 >7
The relationship among [OH-] and [H+] and pH are summarized in the table 3.2

Table 3.2: Values of pH for some common solutions


Solution/ Reagent pH [H+] [OH-]
-13
0.1 M NaOH (Household bleach) 13 1 x 10 1 x 10-1
Household ammonia 12 1 x 10-12 1 x 10-2
Lime water 11 1 x 10-11 1 x 10-3
Borax 10 1 x 10-10 1 x 10-4
Baking powder 9 1 x 10-9 1 x 10-5
Egg white, sea white 8 1 x 10-8 1 x 10-6
Human blood, tears 7 1 x 10-7 1 x 10-7
Milk, saliva 6 1 x 10-6 1 x 10-8
Black coffee 4 1 x 10-4 1 x 10-10
Wine 3 1 x 10-3 1 x 10-11
Cola, vinegar, Lemon juice 2 1 x 10-2 1 x 10-12
Gastric juice 1 1 x 10-1 1 x 10-13
0 1 x 100 1 x 10-14

Point of Emphasis: pH=-log10[H+]. This means that p =- Log10 and this is always the case.
Therefore, pOH= -log10 [OH-]. In any solution [H+][OH-] = 1 x 10-14. By
extension then pH + pOH = 14.

The pH scale in table 3.2 is shown to extend from 0 to 14 because nearly all solutions commonly
encountered have pH in this range. In principle, however, the pH values for strongly acidic
solutions can be less than zero, and for strongly basic solutions, it can be greater than 14.
Practice Exercise 3.2
(a) In a sample of lemon juice, [H+] is 3.8 x l0-4M. What is the pH of this solution?
(b) A commonly available window-cleaning solution has a [H+] of 5.3 x 10-9M. What is
its pH?
(c) A solution formed by dissolving an antacid tablet has a pH of 9.18. Calculate [H+].
(d) Calculate the pH of the following solutions
(i) O.lM H3PO4 (ii) 2M NaOH

3.4.1 The pH of Strong Acids and Strong Bases


The six most common strong acids are HCl, HBr, HI, HNO3, HC1O4 and H2SO4. Thus aqueous
solution of nitric acid, HNO3, consists entirely of H+(aq) and NO-3(aq). This is because the acid
almost completely ionizes as shown in equation 3.8
HNO3(aq) H+(aq) + NO3-(aq) (Complete ionisation)
3.8

In an aqueous solution of a strong acid, the acid is normally the only significant source of H+
ions. As a result, calculations of the pH of a solution of a strong acid is quite simple because [H+]
equals to the original concentration of the acid in case of a monobasic acid. For example, in a
0.20M solution of HNO3, [H-] = [NO3-] is equal to 0.20; the concentration of undissociated
HNO3 molecules is virtually zero.

SAMPLE EXERCISE 3.2

(i) What is the pH of 0.040M solution of HCIO4?

Solution
[H+] = [C1O4-] = 0.040M. Strong acid are completely ionized, hence
pH = -log (0.040) = 1.40.

Practice Exercise
An aqueous solution of HNO3 has a pH of 2.66. What is the concentration of the acid

SAMPLE EXERCISE 3.3


What is the pH of a 0.011M solution of Ca(OH)2?

Solution
[OH-] = 2 x (0.011M) = 0.022M
[H+] = 1 x 10-14/0.022 = 4.6 x 10-13
Therefore
pH = -log(4.6xlO-I3) = 12.34

Alternatively
pOH = - log (0.022) == 1.66
Thus
pH = 14.00 – pOH = 14 – 1.66 = 12.34

Practice Exercise
What is the concentration of a solution of KOH for which pH is 11.89?

3.4.2 pH of Weak Acids and Bases


Most acidic substances are weak acids that only partially ionize in solution. We can express the
extent to which a weak acid ionizes by using the equilibrium constant for the ionization reaction.
We can represent a general acid by the formula HA, where A- is the conjugate base of the acid.
The ionization equilibrium for HA is given by equation 3.9
HA(aq) H+(aq) + A-(aq)
3.9
The corresponding equilibrium expression is given in equation 3.10

[H + ][A − ]
Ka = 3.10
HA

Ka is called the acid-dissociate constant.

Similarly, many substances behave as weak bases in water. Such substances react with water,
removing proton from H2O, thereby forming the conjugate acid of the base and OH+ ions. This is
expressed in 3.11

Weak Base + H2O Conjugate acid + OH-


3.11

The most commonly encountered weak base is ammonia. It ionizes in water as expressed in
equation 3.12

NH3(aq) + H2O(l) NH4+ + OH-


3.12

The equilibrium expression for equation 3.12 is given in 3.13

[ NH 4+ ][OH − ]
K= 3.13
[ NH 3 ][ H 2 O]

Because the concentration of water is essentially constant, the [H2O] term is incorporated in the
equilibrium constant, giving equation 3.14

[ NH 4+ ][OH − ]
K [ H 2O] = K b = 3.14
[ NH 3 ]

The constant Kb is called the base - dissociation constant, by analogy with the acid -dissociation
constant, Ka for weak acids. The constant Kb refers to the equilibrium in which a base reacts with
H2O to form conjugate acid and OH-

The [H+] or [OH-] for weak acids and bases respectively can only be obtained from Ka and Kb
values.

3.4.3: Calculating pH for Solution of Weak Acids


To calculate the pH of O.IM CH3COOH, the following steps are followed. Ka for acetic acid is
1.8 x 10-5
Acetic acid ionizes m solution according to equation 3.15
CH3COOH CH3COO- + H+ 3.15

The equation has the equilibrium expression shown in equation 3.16


[CH 3COO − ][H + ]
Ka = 3.16
[CH 3COOH]
We first note three important points:
i. The concentration of H+ from the dissociation of water is very small, much less than 10-
7
M. This can be neglected
ii. Since the ionisation constant is small the [CH3COOH] at equilibrium will be
approximately equal to the original concentrate i.e 0.1M
iii. At equilibrium, [H+] = [CH3COO-]

Therefore from equation 3.16 we obtain the following calculations


[H + ]2
Ka = = 1.8  10 − 5
[CH 3COOH]o
Hence
[H+]2 = Ka[CH3COOH]
[H + ] = K a [CH 3COOH]o

Therefore

[H + ] = 1.85  10 − 5  0.1 = 1.36  10 − 3 M

Hence
pH = −log 10 (1.36  10 − 3 = 2.87

Alternativ ely
1 1 1 1
[H + ] = K a 2 [CH 3COOH]o 2 = K a 2 0.1 2

Then taking logarithm to base ten on both sides yields


1 1
− log[H + ] = −logK a 2 + −log(0.1) 2

pH = − 1 log(1.85  10 − 5 ) − 1 log0.1 = 2.87


2 2

In general, we use the equation given 3.17 for calculation of pH


pH = 1 pKa + 1 p[acid] 3.17
2 2

PRACTICE EXERCISE 3.4


• What is the pH of 0.01M and1M acetic acid, given that Ka = 1.85x10-5 at 25°C?

For bases, a similar procedure is used but Kb is utilized in place of Ka . Consider the dissolution
of ammonia in water, Kb = 1.79xl0-5 M. We will write the ionization of the base as shown in
equations 3.18 and 3.19

NH3 + H2O NH4OH 3.18


Then

NH4OH NH4+ + OH-


3.19

Using equation 3.19, we can write equilibrium expression for 0.1M NH4OH as shown in
equation 3.20

[NH + −
4 ][OH ]
Kb = = 1.79  10 − 5 3.20
[NH 4 OH]
Making similar assumptions as before, we obtain equation 3.21 as

[OH − ]2
Kb = = 1.79  10 − 5 3.21
[NH 4 OH]o

Hence equation 3.22 for the expression of pOH 3.22

To calculate pH, we do the usual way as shown in equation 3.23

pH = 14 − 2.87 = 11.12 3.23

The general formula for calculating pH of a 1:1 base is given in equation 3.24

pH = 14 − 1 pK b − 1 p[base] 3.24
2 2

Practice Exercise 3.4


• What is the pH of 0.1M CH3NH2 given that the Kb = 5 x 10-4M

• A solution is made by adding solid sodium hypochlorite, NaClO, to enough water to


make 2.00 litres of solution. If the solution has a pH of 10.50, how many moles of
the hypochlorite was added to the water?
3.5: HYDROLYSIS OF SALTS
Salts are products of neutralization of acids by bases. When a salt is added to water the solution
may remain neutral or it may become acidic or basic depending on the type of salt. This is
because certain ions interact with water hence disturbing the self ionisation of water. This
interaction of ions of a salt with water is called hydrolysis.

There are four types of salts

1. Salts of a strong acid and a strong base, for example


HCl + NaOH NaCl + H2O 3.25
The salt dissociates in water as
NaCl + H20 → Na+ + Cl- + H20 3.26
+ -
No interactions occurs between the Na and Cl ions with water. Therefore, there is no
hydrolysis. The solution thus formed is neutral
2. Salts of weak acids with a strong base. Consider
CH3COOH + NaOH CH3COONa + H2O 3.27
The formed salt ionizes in water thus
CH3COONa + H2O CH3COO- + Na+ 3.28

The acetate ion hydrolysis occurs as shown in equation 3.29


CH3COO- + H2O CH3COOH + OH- 3.29
-
Due to the excess OH introduced as a result of hydrolysis, the resultant solution of the salt is
basic

What then is the pH of say 1M CH3COONa solution? Firstly, an equilibrium constant, Kh,
for the hydrolysis may be written ignoring [H2O] as previously explained, thus

[CH 3COOH][OH− ]
Kh = 3.30
[CH 3COO− ]
Based on previous arguments, the [CH3COOH] is equal to [OH-]. We also need to the
hydrolysis of CH3COO- is very low hence [CH3COO-] is equal to 1M. Thus equation 3.30
when correctly substituted gives equation 3.31 as

[OH − ]2
K h = 5.4  10 − 10 = 3.31
1

Proper calculation yields


[OH-] = 2.3 x 10-5
Hence pH is given by

pH =14 – (- log (2.3 x 10-5) = 9.36

Clearly, this salt gives a basic solution as earlier predicted

3. Salts of a strong acid with a weak base. Consider the salts of


HCl + NH4OH NH4Cl + H2O
3.32
The salt dissociates in water to give
NH4Cl + H2O NH4+ + Cl- + H2O 3.33
Hydrolysis of the ammonium ions yields
NH4+ + H2O NH4OH + H+
3.34
Thus the resultant salt solution formed is acidic. The hydrolysis constant expression can be
written as shown in equation 3.35

[NH 4 OH][H + ] K w
Kh = = = 5.6  10 − 10 3.35
[NH 4 OH][H 2 O] K b
Making similar assumptions as in the cases of sodium acetate above, we find that for a 1.0M
NH4Cl solution would be calculated by correct substitution as shown below
[H + ]2
Kh = = 5.6  10 − 10
0.1

[H + ] = 5.6  10 − 10  0.1 = 7.5  10 − 6

Thus we obtain the pH as

pH = −log(7.5  10 − 6 ) = 5.12

As predicted earlier, the NH4Cl solution is acidic

4. Salts of weak acid with a weak base. Consider the following case
CH3COOH + NH4OH CH3COO- + NH4+ + H2O 3.36
Hydrolysis of the resultant salt ions is thus
CH3COO- + NH4+ + H2O CH3COOH + NH4OH 3.37
The hydrolysis constant expression is thus written thus

[NH 4 OH][CH 3COOH]


Kh =
[NH + −
4 ][CH 3COO ]
It is clear that we can relate the expressions of Kh, Kw, Kb and Ka in one equation which
yields equation 3.38

Kw
Kh = 3.38
Ka Kb

Since neither the [H-] nor the [OH-] appears in our hydrolysis equilibrium, equation 3.38, then
pH is dependent on the magnitudes of Ka, and Kb as illustrated below in calculating the pH of a
CH3COONH4 salt solution.
Since the ions are not strongly hydrolyzed then •.
+ -
[NH4 ] = [CH3COO ] = C (concentration of the salt)

More so, [NH4OH] and [CH3COOH] are approximately equal hence

Kw [NH 4 OH][CH 3COOH]


Kh = =
Ka Kb [NH + −
4 ][CH 3COO ]

Hence

Kw
[CH 3COOH] = C
Ka Kb
Since Ka must remain constant in solution, then

[CH 3COO − ][H + ]


Ka =
[CH 3COOH]

Then combining the above equations we obtain

[H + ] =
K a [CH 3COOH] K a Kw
= C
[CH 3COO− ] C Ka Kb

Kw
=
Ka Kb

Thus taking negative logarithm base ten on either sides yields equation 3.39 generally used for
calculation of pH of such solutions

pH = 1 pK w + 1 pK a − 1 pK b 3.39
2 2 2
Clearly, whether the salt is acidic or basic depends on which of the two, Ka or Kb is higher. In
our case, pH = 7 implying the solution is neutral
Practice Exercise 3.5
The dissociation constants of HCN and NH4OH are 7.2 x 10-10 and 1.85 x 10-5. What is
the pH of a 0.05M NH4CN solution?
3.6: BUFFER SOLUTIONS
A buffer solution is a solution whose pH does not change significantly when either H+ or OH-
ions are added to it. They are made by mixing a weak acid with one of its salt with a strong base
or a weak base with one of its salt with a strong base. Acetic acid for example, may be mixed
with sodium acetate (salt with NaOH) in a given ratio to constitute a buffer solution or
ammonium hydroxide with ammonium chloride. Consider a solution prepared by mixing
0.70Macetic acid and 0.6M sodium acetate where Ka(acetic acid) = 1.85 x 10-5M

How can we obtain the pH of the buffer solution and how does it resist change in pH?
The acetic acid is partially dissociated while the salt, sodium acetate is fully dissociated.
Therefore the equillibria 3.40 and 3.41 will exist in solution

CH3COOH CH3COO- + H+ 3.40


- +
CH3COONa CH3COO + Na 3.41
We write the equilibrium expressions for equation3.40 because it is the one giving H+ which
governs the pH of the buffer

[CH 3COO − ][H + ]


Ka = 3.42
[CH 3COOH]
Making [H+] the subject of equation 3.42 yields

[H + ] = K a
[CH 3COOH]
3.43
[CH 3COO− ]

Due to the high [CH3COO-] from equation 3.41, equation 3.40 is suppressed to the left hand side.
Thus we can neglect the dissociation of the acid, therefore in solution, the concentration of the
acetic acid remains as 0.70 M. If this holds, then the concentration of the acetate ions should be
equal to the original concentration of the salt. That is [CH3COO-] = 0.60 M. Substituting the
above information in equation 3.43 yields equation 3.44
[H + ] = K a [
[Acid]
3.44
[Salt]
Taking logarithm to base ten on both sides of the equation 3.44 gives equation 3.45 that is
generally used for calculation of pH of such buffer solutions

 [salt] 
pH = −log[H + ] = pK a + log   3.45
 [Acid] 

Using the above formula to calculate the pH of our buffer yields a pH of 4.66. Certify yourself.
The salt: acid ratio controls the pH of the buffer because Ka never changes. It is important to note
that the salt to acid ratio never changes upon dilution. The pH of the buffer therefore, does not
change upon dilution.

Our second concern was how the buffer resists change in pH. Consider equations 3.40 and 3.41.
If H+ ions are added to a buffer solution, they react with the CH3COO- ions to form CH3COOH
thus concentration of the acid increases while the acetate ions decrease. The salt ioinses to
replace the lost CH3COO-. It is important to note that the [H+] remains constant while the ratio of
the salt: acid however remains approximately constant hence no change in pH occurs. Upon
addition of OH-, the H+ ions react with it, thus more of the CH3COOH ionizes, the concentration
of the acid decreases as the CH3COO- increases. Likewise, the increased CH3COO- combine
with Na+ to form additional CH3COONa thus the [salt] increases slightly. Similarly, the salt: acid
ratio and [H+] remains almost constant thus no significant change in pH occurs.

Sample Exercise
What is the change in H+ ions and hence pH when 1 ml of 1M HCl is added to one litre of
i. Pure water

ii. Buffer solution considered above


3.7: ACID-BASI
Solution
During anuetralization reaction of an acid by abase the pH obviously changes as the H*" ions are
+
i. In
removed puresolution.
from ] = 10-7MpH
water, [HTherefore and pH = 7 can be utilized in determining the end point of
changes
such a reaction. However, we must +take into consideration the hydrolysis of ions at the end point
The number of moles of H ions contained in 1ml of IM HCl is (1 x1)/1000 = 10-3 M.
and therefore the pH at+ this point may be <7, = 7 or >7 depending on the strength of the acid and
Therefore, [H ] changes from 10-7 to 10-3M. This is 104 folds. Thus the pH changes
base used Therefore four cases are encountered and the changes in pH as the- titration continues
from 7 to 3
are represented in figs. 3.1 (a) - (d). +
ii. Again the same amount of H ions are added to the buffer. This reacts with CH3COO_
1. Strong Add - Strong
-3
Base
Consider titration of 25 cm3 of O.Iacid
to form 10 M additional MHCi which
wifeincreases
0.1 MNaOH.from 0.7 to 0.701. The salt ionises to
replace the lostofCH
At the commencement the3COOH
titrationby[H*]
a similar amount.
- 0.1 and ThuspH
therefore the=salt
1. concentration decreases
from 0.6 3 3 3 The pH
After addition of 5tocm0.599M.
NaOH We thus calculate
arc equivalent thecm
of 20 new0.1
buffer
MHCI pHcontained
using equation 3.45.
in 30 cm of
solution is left.
is 4.66. It virtually does not change
20 cm3 0.1 MHCI contain 20/1000 x 0.1 moles'H* ions.
Practice
This amount Exercise 3.6
is now present in 30 cm3 of solution therefore the concentration per liter is
20Calculate1000 the change in pH when 1 ml of 1M NaOH is added to 500 ml of
In casea) ofPure
a weakwater
base with one of its salts with a strong acid, for example NH 4OH with NH4Cl,
similar considerations are made thus equations 3.46 – 3.48 holds
b) TheKabove [base]considered buffer solution
[OH − ] = b 3.46
[acid]
Thus
pOH = pK b + p[base] − p[acid] 3.47
Therefore
pH = pK w − (pK b + p[base] − p[acid]) 3.48
The extent to which a buffer solution resists change in pH is referred to as buffer capacity.
Clearly, when the ration salt: acid in equation 3.45 is unity, we expect the maximum buffer
capacity. Thus this ratio should be kept as close as possible to unity/

3.7: ACID- BASE TITRATION


During a neutralization reaction of acid by a base, the pH obviously changes as the [H +] reduces.
Therefore, pH changes can be utilized in determining the end – point of such a reaction.
However, we must take into account the hydrolysis of ions at the end point and note that some
end – points may occur at pH of 7, <7 or >7 depending on the strength of the acid or base used.
Refer to section 3.5 above for revision purposes. Therefore, four cases are encountered and the
changes in pH as the titration continues are represented I figures 3.1 – 3.4

3.7.1: Titration of a strong acid with a strong base


Consider titration of 25 cm3 of 0.1M HCl with 0.1M NaOH. At the commencement of the
titration, the [H+] is 0.1M and therefore the pH = 1. After addition of 5 cm3 of the 0.1M NaOH,
an equivalent of 20 cm3 of 0.1M HCl contained in 30 cm3 of solution is left. This corresponds to
0.667 M of H+ ions in the solution. This is worked out as follows

20 cm3 of 0.1M HCl contains (20 x 0.1)/1000 moles of H+ ions


This amount is now in 30 cm3 of solution, therefore the molar concentration of H+ ions will be
given by
20  0.1 1000
 = 0.667 moles of H +
1000 30

Thus pH = 1.186
Similar calculations will show that the pH will change as shown in table 3.3

Table 3.3: Corresponding change in pH as 0.1M NaOH is added to 25ml of 0.1M HCl
Vol. of NaOH added 0 5 10 20 24 25 25.1 26
pH 1 1.186 1.37 1.95 2.69 7 10.68 11.29

We should note that since the acid is completely dissociated, initially any added base does not
effect a big change in [H+] thus pH changes gradually before the end point However at the end
point there is a big change as the completely dissociated base will cause the pH to rise rapidly.
Also the pH at the end point is 7 as the Na- and Cl- produced are not hydrolysed. The results are
represented in Fig. 3.1.

3.7.2: Titration of Weak Acid with a Strong Base


Consider the titration of 50 ml 0.1 M acetic acid with 0.1 MNaOH. Initially before any base is
added the pH will be 2.87. Please try and justify this. As the titration progress the [H+] will be
given by
[H + ] =
[CH 3COOH]K a
3.49
[CH 3COO− ]

and therefore pH is dependent on the anion concentration. Calculation of pH changes here as the
titration progresses is rather difficult since the acid is not fully ionised and the degree of
ionisation will change throughout the titration. However at the end point the pH can be obtained
from as in section 3.5.

Curve 3.2 shows how the pH changes during titration. Note that initially there is a marked
increase in pH which is due to the hydrolysis of acetate ions. The end point has a pH greater than
7 as expected and beyond the end point the change is similar to that of strong acid - strong based
as pH now changes only due to the added strong base.

The other two cases represented in 3.3 and 3.4 and can be explained in a similar way.
pH Changes
pH Changes

Figure 3.1: Change in pH for Strong Acid-Strong


Base

Volume of NaOH added


Volume of NaOH added
Figure 3.2: Change in pH for Weak Acid – Strong Base

pH Changes
pH Change

Volume of Weak base added


Volume of weak base added
Figure 3.3: Change in pH for Strong acid – Weak base Figure 3.4: Change in pH for Weak Acid – Weak
Base
3.8: Neutralization Indicators

Indicators are weak organic acids or bases which change colour according to the H+ ion
concentration of the solution or liquid to which they are added. Like any other weak acid or base,
the indicator is partially dissociated i.e. an equilibrium exists between the un-ionised and ionised
forms. For indicators, the un-ionised form has a different colour from the ionised form. Let us
represent an acidic indicator as HIn. We can thus have an equilibrium equation 3.50 thus

HIn H+ + In-
Colour 1 Colur 2 3.50

Colour 1 and colour 2 being two different colours. In an acidic solution, this equilibria shifts due
to the common ion effect to the left hence colour 1, due to me excess HIn, is observed. However,
in a basic solution, OH- ions react with H+ ions to force the equilibrium to shift to the right hence
colour 1, due to the excess, In- ions, is observed. If the colour of the indicator depends on which
of ihe two HIn or In- is in excess, then if a base is added gradually then we expect that the colour
change will be gradual. This is starting with the colour of HIn and as more H+ ions are removed
and to the colour of In- as more of In- ions are formed

3.9: SOLUBILITY EQUILIBRIA

Solubility is the amount of a solute that dissolves in a specified volume of solvent to give a
saturated solution. According to this definition then, few substances are insoluble since even for
salts like silver chloride, which we refer to as insoluble, in a saturated solution there will be
small amounts of Ag+(aq) and Cl-(aq). It is only because these amounts are so small that we refer to
the salt as insoluble otherwise this is not strictly so. Such salts whose solubilities are very low are
said to be sparingly soluble. In such a saturated solution a heterogeneous equilibrium exists. This
is shown in equation 3.51

AgCl(s) Ag+(aq) + Cl-(aq)


3.51

At equilibrium only 1.67 x 10-3 moles AgCl dissolve in 1 litre of water. We can write the
equilibrium constant for the equilibrium thus as given in equation 3.52

a Ag + .a Cl −
K= 3.52
a AgCl

Where ‘a’ is the activity. Since the activity of solids is constant then we can write the above
equation as given in 3.53
K sp = K AgCl = [Ag + ][Cl − ] 3.53

Since ‘a’ is equal to the concentration for dilute solutions. This shows that KSP is just a product
of the concentration of the ions in the saturated solution. It is referred to as the Solubility
Product. Other examples include

CaF2(s) Ca2+(aq) + 2F-(aq)


3.54

La(OH)3(aq) La3+(aq) + 3OH-(aq)


3.55

We should note that under no circumstances can the solubility product be exceeded in a saturated
solution and that for concentrations that yield a product of lower value than Ksp, the salt is
soluble i.e. the solution is not saturated. Since solubility is dependent on temperature then Ksp
also changes with temperature.

Sample exercise 3.7


The solubility of AgCl is 1.67 x 10-5 moles per litre at 25 °C. Calculate me solubility product of
AgCl

Upon complete dissociation of AgCl, one mole of the salt would yield separately 1 mole of Ag+
and Cl- ions. Therefore in the saturated solution,

[Ag+] = [Cl-] = 1.67 x 10-5 moles per litre. Therefore the Ksp for AgCl salt is given as

Ksp(AgCl) = [Ag+][Cl-] = (1.67 x 10-5)2

= 2.8 x 10-10mol2l-2

Practice Exercise 3.7


The solubility of Ca(IO3)2 is 2.18 g/l. Calculate the solubility product of Ca(IO3)2
Solubility from Ksp values
If the solubility product is known, then the solubility of the salt can be calculated as illustrated
from the sample exercise 3.8

Sample Exercise 3.8


Given mat Ksp for AgCl is 2.8xlO-10 mol2l-2 at 25 °C. What is die solubility of AgCl in 1 litre
of water?
Solution
-10 2 -2
Ksp(AgCl) = [Ag+][Cl-] = = 2.8 x 10 mol l

[Ag+] = [Cl-] hence [Ag+][Cl-] = [Cl-]2


+ 2 -10 2 -2
[Ag ] = 2.8 x 10 mol l
Hence
+ -10 2 -2 0.5 -5 -1
[Ag ] = (.8 x 10 mol l ) =1.67 x 10 mol l
This is the concentration of Ag+ present at equilibrium and from the stoichiometry of the
reaction, this is the amount of AgCl(s) that has dissolved. Therefore, the solubility of AgCl(s)
= 1.67 x 10-5 mol I-1
Practice Exercise 3.8
Given tha the Ksp for BaSO4 is 1.08 x 10-10 mol2 l-2, Calculate the solubility in 1 litre
of water

Since KSp cannot be exceeded in a saturated solution any attempt to exceed it by increasing the
concentration of ions results in precipitation so as to maintain K8p. If any of the ions present in
the equilibrium is added from another source, this results in an attempt to increase KSp and
therefore precipitation of the salt occurs. This is in accordance with the Le Charteliers principle.
This is called the Common Ion effect. We can demonstrate this with sample exercise 3.10

Gi ven that Kgp for AgCl as 2.8 x IO-10 M2, determine the solubility of AgCI(s) in 0.1M
AgNO3.

Solution
We can only be able to obtain the solubility of AgCl from the concentration of ions since
there as another source of silver ions. From Ksp equation, we know that
AgCl(s) Ag+(aq) + Cl-(aq)
Thus
Ksp = [Ag+][Cl-] = 2.8 x 10-10M2
In absence of the AgNO3, we expect to have
For a salt that is not univaleat, we should be extra careful in deciding the relationship between
concentrations of the ions in the saturated solution to solubility. Consider the practice exercise
3.11

Practice Exercise 3.11


Given that the Ksp for CaF2 is 1.70 x 10-10 M3, what is the solubility of CaF2 in 0.1 M
Ca(NO3)2?
Solution

We obtain the solubility from the concentration of F- in the saturated solution again due to the
fact that there is another source of Ca2+ ions other than CaF2. Solubility will be equal to 1/2
[F-]. As explained above, we now know that the [Ca2+] will largely be contributed by
Ca(NO3)2 which is 0.1M (we neglect the contribution of CaF2). The Ksp value will not change
thus
Ksp = [Ca2+][F-]2 = 1.70 x 10-10

This implies
(0.1)[F-]2 = 1.7 x 10-10
Hence the [F-] = 4.1 x 10-5M. Going by our argument that the solubility of the CaF2 is ½ ([F-],
then the solubility of CaF2 in 0.1M Ca(NO3)2 is 2.05 x 10-5mol l-1.
Note the contribution of the common ion

-10
We can see that the expected Ksp is lower than 1.7 x 10 M and hence precipitation will not
occur.
In qualitative analysis, say of a suspected Cu2+ and Zn2+ mixture, if we would need to separate
the two ions, We can do so by differential precipitation. Thus, suppose we have a solution
containing 0.02 M each of Cu2+ and Zn2+, how can we separate the two? We are provided with
the information that Ksp for ZnS = 1.00 x 10-2 mol21-1 and for CuS = 8 x 10-37 mol2 l-2. The CuS
is much more insoluble compared to ZnS. Given that

Ksp(H2S) = 1 x 10-22M3, that is [H+]2[S] for the reaction


H2S 2H+ + S2-
3.56
Then due to common ion effect we can control the concentration of S by adjusting the [H+].
-

Thus If we bubble H2S in to a solution of 1M H+ ions, then we will have

[H+]2[S-] = 12[S-] = 1 x 10-22


This implies
[S-] = 1 x 10-22M
If this saturated solution is used in the Zn2+ and Cu2+ mixture, we have then

Ksp(ZnS) = [Zn2+](1 x 10-22) = 1 x 10-2


This implies
[Zn2+] = 1 x 1020M
We do the same for Cu2+ and we obtain that [Cu2+] will be 1 x 10-15M. Clearly, this is an
insignificant amount of Cu2+ ions left in the solution. That way, we will have precipitated copper
from our mix solution. Such a procedure is much utilized in qualitative analysis where ions are
grouped together in inorganic analysis according to their salt solubilities.

SUMMARY

We have seen that acidic solutions are those that contain more H+(aq) than OH-. Basic solutions
contain more OH-. The hydrogen ion is strongly bound to water, for this reason the hydroxium
ion, H3O+is often used to represent the predominant form of H+ in water.

Water spontaneously ionizes to a slight degree (autoionization), forming H+ and OH-. The extent
of ionization is expressed by the ion -product constant for water. At 25 0C, the equation is
represented as

Kw = [H+][OH-] = 1.0 x 10-14

The relationship describes not only pure water, but aqueous solutions as well. Thus, as [H+]
increases, [OH-] decreases.

Strong acids are strong electrolytes, ionizing completely in aqueous solutions. Basic solutions are
produced by ionic hydroxides such as NaOH and by substances that react with water to give
[OH-]. Weak acids are weak electrolytes; only part of the molecules exist in solution in ionized
form. The extent of ionization is expressed by the acid-dissociation constant. A general reaction
can be represented as

HA(aq) H+(aq) + A-(aq)

Weak bases include NH3, amines, and the anions of weak acids. The extent to which a weak base
reacts with water to generate OH- and the conjugate acid of the base is measured by the base-
dissociation constant, Kb. This is the equilibrium constant for the

B(aq) + H2O HB+(aq) + OH-(aq)


where B is the base.

The reaction of ions with water with a resultant change in pH is called hydrolysis. Tlie cations of
strong bases (alkali metal ions and the alkaline earth metal ions) and the .anions of strong acids
do not undergo hydrolysis.

The equilibrium between a solid salt and its ions in solution provides an example of
heterogeneous equilibrium. The solubility product, Ksp, is equilibrium constant that express
quantitatively the extent to which the salt dissolves. Addition to the solution of an ion common
to a solubility equilibrium causes the solubility of the salt to a decrease. This phenomenon is
called the common-ion effect.

Exercise 3

1) The solubility of PbBr2 is 8.4 g I-1 of water. Determine Ksp for PbBr2
2) Ksp for Cu2S is 2.0 x 10-47 m3. Calculate the solubility of Cu2S
3) What is the solubility of BaF2 in 0.20M NaF? K,p BaF2 = 1.7 x 10-6 M3
4) Does CaCO3 precipitate if 50 ml of 3.0.x 10-5 M Ca(NO3)2 is mixed with 50 ml of 4.0x10-
4
M Na2CO3? Ksp (CaCO3) = 8.7 x 10-9 M2
5) What is the percentage decrease in H+ ion concentration corresponding to an increase of
0.1 in pH?
6) What is the hydrogen ion concentration in a solution with a pH of (a) 4 and (b) 2.3?00
7) 10 ml of a 0.21M NaOH solution is titrated with 0.0911M HCI.
a. What is the pH before addition of any acid
b. What is the pH after addition of 12.00 ml HCI
c. What is the pH at the equivalence point
d. What is the pH after addition of 30 ml HCI
e. Determine the volume at the equivalent point.
8) A buffer solution is prepared by adding 0.02 M HCI to 500 ml of 0.02 M sodium acetate
and then making the volume to one litre with water. Calculate the volume of HCI
required to prepare a buffer solution of pH 5. Ka = 1.75 x 105 for acetic acid.
9) Calculate the pH of a mixture of 50 ml acetic acid with 20 ml of 1M sodium hydroxide.
Ka = 1.75 x 1C"5 for acetic acid.
10) In what equation 9 above what will be the ratio salt to acid ratio if the pH of the solution
is changed to 4.3.
UNIT 4: PHASE EQUILIBRIA

4.1 Introduction

A phase is defined as a state of aggregation of molecules of a homogeneous system, i.e. uniform


both in chemical composition and physical state. In unit1 we discussed what makes the three
physical states, gas, liquid and solid different from one another. Depending on the temperature
and pressure the different phases at least of a pure substance can exist in equilibrium. Also,
phases of different chemical composition but same physical state can exist in equilibrium e.g.
liquid/liquid. It is these type of equilibria that we will deal with in this unit. For a pure
substance the various equilibria that can exist at given temperature and pressure can be
represented in what is called a phase diagram e.g. for water, Fig. 4.1 represents its phase
diagram.

NOT SHOWN

Fig. 4.1. Phase Diagram for Water

The solid lines represents the conditions under which any two phases exist in equilibrium.

Thus
AT - the sublimation curve represents Solid - Gas equilibria
TC - the melting curve represents Solid - Liquid equilibria
TTC - the boiling curve represents Liquid - Vapour equilibria

At T, the triple point, all the three phases are in equilibrium. Point TC, the critical point
represent the point beyond which the vapour cannot be converted into a liquid by application of
pressure alone, the temperature must be lowered to below TC for the condensation to occur.

The regions where the different phases exist are indicated on the diagram. With the aid of a
phase diagram therefore we can be able to describe precisely the state in which a given substance
will exist at any given temperature and pressure.

4.2: LEARNING GOALS


By the end of this unit you should be able to:
• Describe, with the aid of a phase diagram, the state in which a given substance
will exist at any given temperature and pressure
• Explain the causes of deviation from ideal behaviour for solutions
• Describe the effect of solute concentration on the vapour pressure of a two
component solution.
• Use Raoult’s Law to solve for one variable (Pi, Xi Poi ) given the other two
variables or information from which they can be determined.

4.3: LIQUID – VAPOUR EQUILIBRIA

At any given temperature below the boiling point of a liquid (or liquid mixture) there exists a
vapour, exerting a vapour pressure over the liquid in equilibrium. Indeed the liquid boils when
this vapour pressure, which increases with temperature becomes equal to the external
(prevailing) pressure. This liquid - vapour equilibria is not very important for a pure substance
but becomes interesting in liquid - liquid mixture. Some pairs of liquids (we shall limit our
discussion to pairs) completely mix, and are said to be miscible, while others do not mix, hence
immiscible and others still are partially miscible. Let us consider miscible liquids first. These
may form an ideal solution in which the cohesive forces are just the same as those existing in the
separate components of the solution i.e. forces existing in a solution of A and B are just the same
as those existing in pure A and pure B.

Most variable composition equilibria are represented either at Constant T with P and
Composition as the variables or at Constant P, with T and Composition as the variables.

[A solution is any homogeneous phase that contains more than one component. The component
that constitutes the large proportion is referred to as the solvent and that of lesser proportion the
solute].

Is there an ideal solution?

We look for what we may call an ideal solution with a hope that the concept may lead us to a
better understanding of solutions in general.

Imagine a solution of molecules A and B.

If molecular sizes are the same and intermolecular attraction of A for A and B for B are the same
as the attraction of A for B. The solution is considered ideal when there is a
complete uniformity of intermolecular forces.

4.3.1: RAOULT'S LAW

Partial vapour pressure measures the escaping tendency of a molecule from solution, which in
turn is a measure of the cohesive forces present in solution. This law, discovered experimentally,
is applicable to liquids miscible in all proportions.
Raoult’s Law: It states that at a given temperature the partial vapour pressure of
component I in a liquid mixture is Pi = Pi0 XI where XI is the mole
fraction of that component in the liquid phase, and Pi0 is its saturated
vapour pressure.

Sample Exercise 4.1

At 880C the saturated vapour pressure of benzene and toluene are 953 and 378 mmttg
respectively. Calculate the vapour pressure of a benzene-toluene mixture containing 2 mol of
benzene per mole of toluene, assuming that Raoult's law is obeyed.

Solution:
Here the mole fraction of benzene (XB) and toluene (XT) are 0.66 and 0.33 respectively.
Therefore P = PB + PT = PB0 XB + PT0 XT = (953 x 0.67) + 378 x 0.33) = 763 mmttg.

Practice Exercise 4.1

NOT SHOWN

4.3.2: VAPOUR PRESSURE OF HOMOGENEOUS (TWO COMPONENTS) BINARY


SOLUTION

If each component of the liquid mixture obeys Raoult's law then P1 = P10 X1 and
P2 = P2o (1 - X1). Therefore, at constant T, the total vapour pressure is P = P1 + P2
= P20 + X1 (P1o - P20) P = P10 X1 + P20 (1 - X1)
= P20 + X1 (P10 - P20)

The changes in the total and partial pressures with composition at constant temperature are
represented as below:
4.3.3: DISTILLATION OF BINARY SOLUTIONS

A homogeneous liquid mixture boils when the sum of the partial vapour pressure of its
components equal to the external pressure. In most cases the composition of the vapour differs
from that of the boiling liquid.

If the solution is ideal (i.e. obey Raoult's law) then the composition of the liquid boiling at a
particular temperature is obtained from

Composition of liquid P = P20 + X1 (P10 - P20)

The corresponding vapour composition in terms of mole fraction of component 1 in the vapour
phase, is obtained as follows:

Composition of vapour
0
P1 P X
y1 = = 1 1
P1 + P2 P

The above equations relate the liquid and vapour composition at the temperature at which the
total vapour pressure is P. The results of such calculations for liquids of various composition at
constant P can be plotted on a "boiling-point-composition" diagram.

NOT SHOWN

It will be noticed that for systems of this type the vapour is always richer than the liquid in the
more volatile component.

Binary solutions giving boiling-point diagrams similar to the one above can be separated into
their components by successive distillation.
Let us suppose that the original solution has a composition represented by point a. The solution
will boil at a temperature corresponding to b, and give a vapour of composition c, which
condenses to give a liquid of the same composition d etc. Therefore a series of volatisation and
condensation along b-c-d-e-f-g etc, finally produces the pure vapour of the more volatile
constituent.

The volatisation - condensation steps described above can be performed in one operation by
using a fractionating column.

In it's simplest form the column consists of a tube loosely packed with a suitable solid material
(e.g. glass beads) and vertically attached to the top of the distillation flask. When the mixture in
the flask is boiled, the vapour condenses at various levels in the fractionating column. The
resultant liquid flows back down the column and is re-volatilised by the upcoming vapour, so
that the a-b-c-d - etc take place automatically. The technique of fractional distillation is of
immense important both in the Laboratory and in various industrial processes (e.g. the refining of
petroleum).

4.3.4: DEVIATIONS FROM RAOULT'S LAW

Deviations arise when the two components (1 and 2) of a binary solution do not exhibit similar
molecular interactions. As a result of these deviations the vapour pressure - composition
diagram is no longer linear, and this in turn affect the shape of the boiling point - composition
diagram. We distinguish two types of deviation.

1) Negative Deviation

This occurs when the attraction between component 1 and 2 is greater than the attraction 1-1 and
2-2. This may be visualized as the holding back of molecules that would otherwise go into the
vapour state. There is a reduction in the vapour pressure of the system. If the deviation is
sufficiently large, the vapour pressure diagram exhibits a minimum, and the boiling point
diagram exhibits a maximum.

Consequently, it is no longer possible to completely separate the components of the system by


fractional distillation. [Consider for example a mixture of initial composition a. Successive
volatisation and condensation will proceed along a-b-c-d - etc so that component 2 will be
preferentially volatilised and the residue in the flask will tend towards composition X, after
which further volatilisations and condensations will produce no further change in composition.

Similarly, if the original mixture is of composition p, the the process will occur along
p-q-r-s- etc and component 1 will be lost while the residue tends towards X. The liquid of
composition X has the highest boiling point and is referred to as an azeotropic or "constant
boiling" mixture. Such mixtures can easily be distinguished from compounds, because their
composition vary with pressure.

Examples: Chloroform/acetone
Cl3CH - - OC(CH3)2 hydrogen bonding
hydrogen chloride/water HCl + H2O → H3O + Cl -

NOT SHOWN

Positive Deviation

This type of deviation occurs when the attractive forces between 1-1 and 2.2 are of similar
magnitudes, but the attraction 1-2 is weaker, so that each constituent vaporises more easily in the
presence of the other. The vapour pressure of the system is greater than expected.

The result of a large positive deviation

NOT SHOWN

The vapour pressure diagram exhibits a maximum and the boiling-point diagram a minimum.
The distillate composition tends towards Y, a minimum-boiling azeotrope, whereas the residue
becomes richer in either component 1 or 2.

Ethanol/Water at 1 atmosphere 95.6% ethanol boils at 78.20C.

i.e., Fractional distillation cannot completely remove water from aqueous ethanol.

When positive deviations from Raoult's law is sufficiently large, instead of exhibiting azeotropy
the two components may become only partially miscible (eg. phenol and water) or virtually
immiscible (water and chlorobenzene)

4.4: IMMISCIBILITY

Provided a system composed of two immiscible liquids is adequately agitated, so that each liquid
has a chance to vaporise, the total vapour pressure of the system will be the sum of the saturated
vapour pressures of the two components separately.

P = P10 + P20

The system boil at a temperature lower than the boiling points of the two components. This
phenomenon is exploited in the technique of steam distillation whereby a substance immiscible
with water may be distilled at a temperature lower that 1000C by blowing a current of steam
through it. After condensation, the substance and water form two layers which may be
separated.

W1 and W2 are masses of the two substances in the distillate, M1 and M2 being their molar
masses and n1 and n2 being the number of moles co-existing in the vapour phase, assuming ideal
behaviour.

0
WeightofX MxPx
=
Weightofwa ter M H 2 0 P V H 2 O

At any given pressure P the boiling point of the system may be predicted by plotting P10 and P20
against T1 and finding the temperature at which P10 + P20 = P.

Sample Exercise 4.2


For n-octane (b.pt = 1260C, M1 = 114x10-3kg mol-1) the steam distillation temperature at 760
mmttg is found to be 900C. At that temperature the saturated vapour pressure of water (P20) is
526 mmttg. How much n-octane will distill over per unit mass of water?

exploited in steam distillation


a substance immiscible with water may be distilled at a temperature lower than 1000C by
blowing a current of steam through it. After condensation, the substance and the water form two
layers which may be separated.

W1, W2 masses of two substances in the distillate

M1 and M2 molar masses and n1 and n2 being the number of moles coexisting in the vapour phase
at any given time

boiling point is the T at which P10 + P20 = P

4.5: PARTITION OF SOLUTE BETWEEN TWO IMMISCIBLE SOLVENTS

When a solute (i) is able to dissolve in two immiscible solvents (A and B) in contact, it will
distribute itself between them so that at equilibrium.

(X i ) A
= KD
(X i ) B

X - mole fraction

where KD is the distribution coefficient

For dilute solution XI is proportional to concentration C

(Ci )A
= KD
(Ci )B

This is the basis of liquid-liquid extraction

Eg. a solute (in water is removed by shaking the solution with a immiscible solvent (e.g. ether)
and then separating the two layers.

- Note that, a fixed volume of second solvent will extract more solute if it is used in several
small quantities rather than all at once in a single extraction.
UNIT 5: ELECTROCHEMISTRY

5.1: INTRODUCTION
Electrochemistry is the branch of chemistry that deals with the relationship between electricity
and chemical reactions. The transfer of electrons that occurs during oxidation-reduction reactions
can be used to produce energy in the form of electricity. In other instances, we can use electrical
energy to make certain non-spontaneous chemical processes occur (Electrolysis). We shall begin
our study of electrochemistry by learning the simple galvanic or voltaic cell, labeling the anode,
the cathode and directions of ion and electron movement and the signs of the electrodes and later
discuss briefly electrolysis.

5.2: LEARNING GOALS


By the end of this unit you should be able to:
• Define a galvanic cell
• Represent a cell on a diagram
• Write the anode, cathode and cell reaction for a given cell
• Given appropriate electrode potentials, calculate the emf generated by galvanic cell
• Describe the Hydrogen electrode
• Define Standard electrode potential
• Given appropriate electrode potentials, calculate the emf generated by galvanic cells
• Define electrolysis
• Predict products of simple electrolytic cells

5.3: AN ELECTROCHEMICAL CELL


When a piece of zinc is placed in a solution containing Cu2+ (aq) ions, some of the zinc dissolves
and copper is plated onto the zinc bar. There is a spontaneous “tendency” for this elecron transfer
reaction to take place:

Zn(s) → Zn2+ (aq) + 2e- oxidation


Cu2+(aq) + 2e- → Cu (s) reduction

Zn (s) + Cu2+ (aq) → Zn2+ (aq) + Cu (s) overall reaction

Kc for this reaction is very high. At equilibrium the product Cu and Zn2+ predominate over the
reactants. There is a greater tendency for copper ions to be reduced (gain electrons) than for zinc
ions to be reduced. This tendency has been tabulated as a series of reduction potentials.

If the tendency for the zinc to give up electrons to copper ions were utilized so that electrons
were made to flow through an external circuit, useful electrical work could be done. This is
known as the voltaic or galvanic cell.
Figure 5.1

Electrons flow from the zinc elecrode to the copper electrode. The zinc electrode is negative and
the copper electrode is positive. Oxidation takes place at the anode and reduction at the cathode.

An electrochemical cell or galvanic cell consists of

1. Two conductors (electrodes) immersed in electrolyte solutions


o anode = electrode at which oxidation occurs

Zn(s) → Zn2+ (aq) + 2e-

o cathode = electrode at which reduction occurs

Cu2+(aq) + 2e- → Cu (s)

2. A salt bridge (to avoid mixing) - in this case KNO3 In order that a charge disparity
produced by the movement of electrons during the operation of the cell can be neutralized
and the electroneutrality maintained eg: in the zinc-copper cell, Zn2+ are produced at the
anode and Cu2+ consumed at the cathode. An overall positive charge in the solution at the
anode and an overall negative charge in the solution at the cathode would prevent further
movement of electrons and the cell would cease to function. The salt bridge allows
migration of ions: anions towards the anode and cations towards the cathode. conduction
of electricity from one electrolyte solution to another - occurs via migration of K+ from
the bridge in one direction and NO3- in the other

3. Voltage measuring device - potential difference is developed by this cell, this potential
difference is a measure of the driving force of the reaction = potentiometer - two
connectors are positive and negative (indicates direction of current) Seen as = if the
concentrations of reactants and products in the cell are far from equilibrium value,
potential difference is very large. If concentration of reactants and products are at
equilibrium, potential difference = 0

Points of Emphasis

1. If concentrations are far from equilibrium, current flowing through circuit can be
harvested - the principle of battery

Battery - electrochemical cell from which we receive energy

Galvanic or voltaic = store electricity - spontaneous chemical reaction to generate


electricity

2. We can also put a battery or power supply across cell and drive the electrochemical
reaction - electrolytic cell = requires an external source of electrical energy - energy
required because we force the reaction to proceed in the opposite direction
3. In a galvanic cell, the anode has a negative charge, and the cathode has a positive charge

Teaching Note: An aid to remembering the signs of the electrodes in a galvanic cell is the
“an” in anode stands for “a negative electrode”

5.4: REFERENCE ELECTRODES AND STANDARD REDUCTION POTENTIALS


A comparison of the “reduction tendency” of various substances can be made. It would be useful
if the potentials for different half-cells could be tabulated. However, the potential of the half-cell
itself cannot be measured: it is possible to measure only the potential difference between two
half-cells. We therefore select arbitrarily a reference electrode relative to which all other half-
cells are measured. The results are tabulated for standard half cells, that is, all reacting species
are at a concentration of 1 M (strictly speaking, at unit “activity”) if in solution or at constant
pressure of 1 atmosphere if a gas.

The Standard hydrogen electrode – SHE


The standard half-cell chosen as the reference half-cell is the standard hydrogen electode; it
contains a platinised platinum electode (inert) dipping into a solution of HCl which is 1 M in
H+(aq) and having H2 gas at 1 atmosphere pressure continually bubbled over the electrode. The
reduction reaction:

H+ (aq) + e- → ½H2(g)

Is assigned arbitrarily a value of 0.00 V, i.e. Eo = 0.00 V at a selected temperature, usually 298
K: Eo is the standard reduction potential, standard indicative relative to the hydrogen half-cell
described above.
The standard reduction potential for Zn2+ (aq) can be measured by setting up an electrochemical
cell which contained H2/H+ aq) (1 M ) in one half-cell and Zn/Zn2+ (aq) (1 M) in the other and
measuring the emf of the cell (all solutions being at the same temperature, usually 298 K). The
emf of the cell is then the standard reduction potential for the reduction reaction

Zn2+ (aq) + 2e → Zn(s) E = -0.76 V

E values for other half-cells can be determined in this way and tabulated. Notice that the
concentrations of the solution in each half-cell is 1 M in the determination of E values. If it is
not 1 M, the value determined is not the Evalue but some other value which depend on the
concentration of the solution.

Detailed tables of E values at 298 K are given in text-books and SI chemical Data.

The more positive the value of E, the greater the tendency for reduction and the better the
oxidizing agent. The more negative the value of E, the lower the tendency for reduction and the
better a reducing agent is the reduction product.

Representation of Cells

Line Diagram: Zn Zn2+(aq) (1 M) Cu2+(aq) Cu

(i) the half-cell of the anode, where oxidation occurs, i.e. the half cell
with the lower reduction potential, is written on the left: the
concentration of the species present is shown in bracket
(ii) the double vertical lines represent the salt bridge when the half-cell are
not in contact (as in Figure 5.1). The ions or aquated species of the
half-cell should be next to the salt bridge symbol
(iii) the half-cell of the cathode, in which reduction occurs, i.e. the half-cell
with the more positive E (less negative) is written on the right.

Calculation of E Values

The E (emf) of the cell is calculated from

Ecell = Eright - Eleft


Where the E values are both relative to E H2H+ (1 M) being assigned a value of 0.00 V. Notice
that TWO STANDARD REDUCTION POTENTIALS ARE INVOLVED

Eg: for the above cell Ecell = 0.43 – (-0.76) = +1.10 volts

If the cell is correctly written with the anode on the left and cathode on the right and E for the
cell (concentrations 1 M) is calculated as above, then E  0 (positive) and the redox reaction
proceeds in the direction shown, that is, zinc displaces copper ions from solution. A cell so
constructed will produce a voltage of 1.10 V. If E  0 (negative), the spontaneous redox
reaction proceeds in the manner opposite to that written. The spontaneous reaction can be
reversed if a voltage greater than the cell voltage is applied to the cell.

5.5: THE NERNST EQUATION

What factors determine the magnitude of the cell potential?

• Energy is the inherent driving force of reactions. All reactions move to minimize their
energy, so if a lot of energy is released when the reaction occurs, the potential will be
large. This is expressed as the standard electrode potential, Eo.
• Concentration. If concentration is high , Le Chatelier's principle would say the driving
force is greater, therefore the potential is greater

These concentration and electrode potentials are taken into account using the Nernst Equation:

Given a hypothetical reaction:


Ox + ne- ---> Red
Half cell potential associated with this half cell is given by:
E = E0rxn - (RT/nF)ln{[Red]/[Ox]}
0
where E rxn is the thermodynamic driving force for the reaction when the other half-cell of the
reaction is SHE
NOTE: All of these E values are based on writing the equation as a reduction (this is also
known as the standard reduction potential)
Cl2 + 2e- ---> 2Cl- E0 = 1.36 V Strong tendency to occur as a reduction (i.e. as written)

Na+ + e- ---> Na E0 = -2.71 V Strong tendency to occur as an oxidation (i.e. opposite direction
as written)
If we take the Nernst equation, convert it to base 10, put in the actual values for R and F and
assume we are at room temperature (298K), we get:
E = E0rxn - (0.059/n)log {[Red]/ [Ox]}

Sample Exercise 5.1:


1. Calculate the half-cell potential for Zn electrode in 10-2 M Zn(NO3)2
half-cell reaction: Zn2+ + 2e- ---> Zn
E = E - (0.059/n) log 1/[Zn2+]
E = -0.763 - 0.059/2 log {1/0.01}
E = -0.822 V

Note:
The reaction proceeds as an oxidation, therefore Zn is the anode
This is the cell potential we would get if we assembled the cell:
Any half-cell potential calculated using the Nernst Equation is an E for a hypothetical cell which
has the half cell of interest as the cathode and the SHE as the anode. However, the negative sign
on our example indicates that if assembled, the Zn electrode would be the anode.

Consider the half – cell consisting of the SHE, but P(H2) and [H+] are different! [P{H2} = 0.5
atm and [H+] = 0.01 M]

So, reaction is still:


2H+ + 2e- ---> H2

E = E - 0.0059/2 log {PH2/[H+]} = 0 - 0.059/2 log [0.5/(0.01)2]


E = -0.1093 V

This is how you calculate the half- cell potential, but we would also like to do this for the whole
cell (i.e. when you actually have two half-cells connected to each other).
So, in order to calculate Ecell

• write both half-cells reactions as reductions


• calculate Ecell = ER – EL using Nernst equation
• Ecell = ER – EL

Sample Exercise 5.2


Calculate the Ecell for the following :
Zn /ZnSO4(1.0 M)// CuSO4(1.0 M)/ Cu
anode on left:
Zn2+ + 2e- ---> Zn(s) E = -0.763 V

Ean = -0.763 -0.059/2 log [1/[Zn2+]]


Eleft = -0.763 V
cathode on right
Cu2+ + 2e- ---> Cu(s) E = 0.337 V
Eright = 0.337 -0.059/2 log (1/1.0)
Eright = 0.337 V
Ecell = Eright - Eleft = 0.337 - (-0.763) = 1.10 V
So, cell reaction is spontaneous as written, the anode reaction is in fact the oxidation and the
cathode reaction is in fact a reduction reaction
So, what if the cell were drawn in this way:
Cu /CuSO4(1.0 M)// ZnSO4(1.0 M)/ Zn
Cu is now anode, Zn is cathode - if you were to follow the same steps, you find that the E for the
cell would be negative. Try this!
Sample Exercise 5.3

Calculate the electrode potential for a Cd electrode in a solution of 0.0100 M Cd2+

Cd2+ + 2 e- = Cd (s) E° = -0.403 V


E = E° - (0.05916/n) log([B]b/[A]a)
E = -0.403 - (0.05916/2) log(1/0.0100)
E = -0.462 V

Sample Exercise 5.4


Calculate the potential for the following cell
Zn | ZnSO4 (5.00 x 10-4 M) || PbSO4 (sat'd) | Pb

Step 1. PbSO4 (s) + 2 e- Pb (s) + SO42- (aq) E° = -0.350 V


2+ -
Zn (aq) + 2 e Zn (s) E° = -0.763 V

Step 2. E = E° - (0.05916/n) log([B]b/[A]a)


E+ = E° - (0.05916/2) log (5 x 10-4)
E+ = -0.252 V

Step 3. E- = -0.763 - (0.05916/2) log (1/5 x 10-4)


E- = -0.860 V
Step 4. Ecell = -0.252 V - (-0.860 V) = + 0.608 V

5.6: E° and the Equilibrium Constant


The equilibrium constant for the reaction:
aA+ bB = cC +dD is written as

a cC a dD
Keq = a b where a = activity (concentration)
a Aa B

Therefore the associated Nernst equation can be written thus

RT
E cell = E o − InK eq
nF

When a cell is at equilibrium E=0 hence


RT
Eo = InK eq
nF

Therefore we can calculate Keq from E values.

Sample Exercise 5.6


Calculate the equilibrium constant for the reaction:

PbPb2+ (0.1 M) Ag+ (0.01 M) Ag

E = 0.93 V

0.0257
Then 0.93 = InK eq
2

Therefore In Keq = 72.37

Keq = 2.7 x 1031


Note the high value of Keq since the reaction will go to completion

Sample Exercise 5.7


A piece of copper is placed in a 0.050 M solution of AgNO3. What is the equilibrium constant of
the solution? The reaction is: Cu (s) + 2 Ag+ Cu2+ + 2 Ag (s)
+ -
Half-reactions: 2 Ag + 2e 2 Ag (s) E° = 0.799
Cu2+ + 2 e- Cu (s) E° = 0.399
E°cell = E+ - E- = 0.799 V - 0.339 V = 0.460 V
K = 10(2(0.460)/0.05916) = 1015.55 = 3.55 x 1015

Electrolysis
We have just seen how chemical reactions produce electricity. The reverse can also happen in
which case electricity may cause chemical reactions to take place.
When a current flows through a solution not only is heat evolved but a chemical reaction also
takes place, the solution being decomposed by the current flow. This phenomenon is known as
electrolysis. A solution which is capable of being decomposed electrolytically is known as an
electrolytic solution and the solute is known as an electrolyte.

Teaching Note: Electrolysis is the ‘opposite’ of reactions of galvanic cells

Electrolytic Cell
An electrolytic cell consists of two electrodes, anode and cathode, dipped in a electrolytic
solution. The electrodes are connected to an external power source (a battery or power supply).
The anode (+ve) is connected to the positive terminal while the cathode (-ve) is connected to the
negative terminal.

Note that this is the opposite of the galvanic cell

In some cases the electrodes are attacked or take part in the reaction during electrolysis, e.g Zn
electrode or Cu electrode in ZnSO4 or CuSO4 solutions, respectively. In most cases electrodes
are inert and they only participate in the exchange of electrons. The chemical changes occurring
involve only the electrolyte.
Consider the electrolysis of an aqueous solution of copper chloride using carbon electrodes.
When copper chloride crystals are dissolved in water they break to form Cu2+ and Cl- ions.
Note that there will also be H+ and OH- ions but ignore these for the moment

NOT SHOWN

Fig. 6.3 An electrolysis cell with inert electrodes.

Since the electrodes are inert (do not take part in the reactions), on passage of
sufficient electric current, the following reactions take place.
At the positive electrode which attracts negative ions , the Cl- ions are discharged on
reaching the electrode. They loose the electrons according to the reaction.

Cl- ------------ > 1/2Cl2 + e [6.15]

This is an oxidation reaction. (Loss of electrons)

Teaching Note: As in the galvanic cell oxidation again takes place at the anode
The electrons, given up at the electrode, travel in the outer circuit to the cathode. At the cathode,
the Cu2+ ions reaching the electrode gain the electrons and are discharged (lose charge), by
gaining electrons and therefore reduced i.e.

Cu2+ + 2e -------->Cu [6.16]

Each Cu2+ requires 2 electrons in the reduction process. Hence 2 Cl- ions must be oxidized (or 2
Cl atoms must be formed)

2Cl- ----------> Cl2 + 2e

The two equations can be written and added to get an overall equation as shown below:

Cu2+ + 2e ---------->Cu Cathode reaction

2Cl- -----------------> Cl2 + 2e Anode reaction


------------------------------------
CuCl2 ---------------> Cu+ Cl2 Overall cell reaction

(Note that the two electrons appearing on both sides of the equation ‘cancel’ out).
The ease with which a substance gets reduced as presented in the electrochemical series
and the ease with which anions get oxidized determines which ions are discharged at the
electrodes. The more easily the cation is reduced and the more easily the anion is oxidized the
easier it will be for them to be discharged
Thus hydrogen is evolved in place of Zinc in a solution of zinc sulphate due to the
presence of H+ ions and oxygen in place of sulphate ions (See the electrochemical series)
If the electrodes are not inert, i.e. they participate in the reactions then they too must be
taken into account in determining ease of oxidation and reduction
Consider electrolysis of copper (II) sulphate solution using copper electrodes.
At the cathode, the Cu2+ ions reaching the electrode are reduced (or discharged) to form
Cu atoms (In preference to H+ ions). These Cu atoms are deposited on the copper electrode and
hence the cathode increases in mass. On the other hand, at the anode we have SO 42-, OH- ions
reaching the anode available for oxidation as well as the copper electrode which is also capable
of oxidation. Among the three Cu is the most easily oxidized therefore it gives up electrons to
yield Cu2+ ions, i.e. the electrode is ‘dissolves’ by going into solution as Cu2+ ions.

Anode: Cu(s) -------------------> Cu(aq) 2+ + 2e

Cathode: Cu2+ (aq) + 2e ----------------> Cu(s)


For every Cu atom that goes into solution as Cu2+ ion there is a Cu2+ ion that is reduced and
deposited as Cu atom on the cathode. Hence the net result is that on average the concentration of
copper (II) sulphate remains unchanged, but the cathode gains mass and the anode is eaten away,
losing a mass equal to that gained by the cathode.

Summary:
1. Oxidation reaction takes place at the anode
2. Reduction reaction takes place at the cathode.
3. The discharge of the ions at the electrodes depend on the positioning in the
electrochemical series

Laws of Electrolysis
1. The amount of primary product formed at an electrode during electrolysis is
proportional to the amount of electricity passed through the cell. Amount of
electricity Q = i x t where i is current in amperes and t is time in seconds.
This is because the amount of electricity is a measure of the no. of electrons available to take
part in the reaction
2. The amounts of various products formed at an electrode by the passage of the same
quantity of electricity during electrolysis are proportional to their relative molecular
or relative atomic mass divided by the change in their charge numbers accompanying
the chemical reaction.
This is because discharge is dependent on the number of electrons required for the process
which in turn depends on the charges involved

SUMMARY
Spontaneous oxidation – reduction reactions can be used to generate electricity in galvanic cells.
The electrode at which oxidation occurs is called the anode, and the electrode at which reduction
occurs is called the cathode.

A galvanic cell may be thought to possess a “driving force” that moves the electrons through the
external circuit, from anode to cathode. This driving force is called the electromotive force (emf)
and is measured in volts. The emf of a cell can be regarded as being composed of two parts: that
due to oxidation at the anode and that due to reduction at the cathode (Ecell = Eox + Ered).

Oxidation potentials (Eox) and reduction potentials (Ered) can be assigned to half-reactions by
defining the SHE as a reference:

2H+(1M) + 2e- →H2(1 am) E = 0.00 V

Standard reduction potentials are referred to as Standard electrode potentials and are tabulated
for a great variety of reduction half reactions.

The Nernst equation relates emf under non-standard conditions to the standard emf:

2.303RT
E = Eo − log Q
nF
At equilibrium, Q = K and E = 0, so that

2.303RT
Eo = log K
nF

which relates the standard emf to the equilibrium constant K.


EXERCISE FIVE
1. What are the standard electrode potentials and the cell reactions when (a) Zn
electrode and (b) Cu electrodes are connected to the hydrogen electrode

2. Consider the cell made up of Cu/Cu2+ and Ag/Ag+ electrodes


a) Represent the cell on a linear diagram
b) Write the half cell and cell reactios
c) Determine the emf of the cell
3 For the electrochemical cell represented by
(Pt)Fe3+(aq),Fe2+ (aq) Br- (aq), Br2 (aq) (Pt)
Write the ion-electron half-reactions for reduction at each electrode, and obtain values for
E from SI Chemical Data
Which reduction will be spontaneous? (i.e. which is the cathode reaction?)
What is the anode reaction?
What is the cell reaction?

You might also like